Categories
Berkeley Carnegie Institute of Technology Chicago Columbia Cornell Duke Economist Market Harvard Illinois Indiana Iowa Johns Hopkins M.I.T. Michigan Minnesota Northwestern Princeton Salaries Stanford UCLA Virginia Wisconsin Yale

Economics Faculty Salaries for 15 U.S. universities. Hart Memo, April 1961

 

Here we have a memo written by member of the Columbia University economics department executive committee, Albert G. Hart, that presents the results of what appears to be his informal polling of the chairpersons of 21 departments. Fifteen of the departments provided the salary ranges at four different ranks. No further details are provided, this one page memo was simply filed away in a folder marked “memoranda”. Maybe there is more to be found in Hart’s papers at Columbia University. Up to now I have only sampled Hart’s papers for teaching materials and perhaps next time, I’ll need to look into his papers dealing with departmental administrative affairs.

For a glance at salaries about a half-century earlier:  Professors and instructors’ salaries ca. 1907

________________

AGH [Albert Gailord Hart] 4/21/61

CONFIDENTIAL information on economic salaries, 1960-61, from chairmen of departments

Institution

Professors Associate professors Assistant professors

Instructors

Harvard

$12,000-22,000

$9,000-12,000 $7,500-8,700

$6,500

Princeton

$12,000-…?…

$9,000-11,500 $7,000-8,750

$6,000-6,750

California

$11,700-21,000

$8,940-10,344 $7,008-8,112

$5,916-6,360

MIT

$11,000-20,000

$8,000-11,000 $6,500-9,000

$5,500-5,750

Minnesota

$11,000-18,000

$8,500-11,000 $6,800-8,400

?

COLUMBIA

$11,000-20,000

$8,500-10,000 $6,500-7,500

$5,500-5,750

Northwestern

$11,000-…?…

$8,000-11,000 $6,800-7,500

?

Duke

$11,400-16,000

$8,200-10,000 $7,200-8,200

$5,800-6,500

Illinois

$11,000-15,000

$7,500-10,000 $6,900-8,600

$6,500-7,100

Cornell

$10,000-15,000

$8,000-10,000 $6,500-7,500

$5,500-6,500

Indiana

$10,000-14,800

$8,300-10,000 $6,500-7,500

?

Michigan

$10,000-…?…

$8,700-..9,500 $6,600-8,000

$5,000

Virginia

$..9,800-15,000

$7,800-..9,800 $6,600-7,800

?

Wisconsin

$..9,240-16,150

$8,000-..9,000 $6,550-8,460

$5,250-5,450

Iowa State (Ames)

$..8,500-13,000

$7,500-..8,500 $6,700-8,000

$4,700-6,600

[…]

Note: The following institutions for which data were not included in the source materials are believed to pay their economists at scales at or above the Columbia level:

Carnegie Tech
Chicago
Johns Hopkins
Stanford
Yale
UCLA

[…]

 

Source:  Columbia University Archives. Columbia University, Department of Economics Collection. Carl Shoup Materials: Box 11, Folder: “Economics—Memoranda”.

Categories
Chicago Economists

Chicago. Economics Department on Possible Candidate for Permanent Employment, 1950

 

How big was the split within the department of economics in 1950 at the University of Chicago? Judging from the decision by chairman T. W. Schultz to essentially table the matter of approaching the central university administration with a candidate for a permanent position, there was a departmental deadlock.

The half-dozen economists discussed were: George Stigler, Abba Lerner, Kenneth Boulding, Leonid Hurwicz, Kenneth Arrow, and Lawrence Klein. Contemplate those names for a moment and then read aloud the following two sentences:

Several members of the Department stated that none of these men had all of the qualities sought: a good mind reaching out fruitfully in new directions in economics. It was agreed, however, that there were no likely candidates possessing these qualities in a high degree.   

We can only speculate which alpha economists happened to lock horns in those three meetings.

_________________________

From the MINUTES, Meeting of the Department,
May 24, 1950.

Present: T. W. Schultz, T. Koopmans, A. Rees, H. G. Lewis, D. G. Johnson, E. J. Hamilton, R. Burns, J. Marschak, F. H. Harbinson, F. H. Knight, M. Friedman, B. Hoselitz, L. Metzler

[…]

II. Appointments

Schultz informed the Department that Hildreth’s position has been renegotiated for a term of three years. The Department approved a motion authorizing for Hildreth the courtesy rank of Associate Professor for a three year term.

The Department then considered the appointment problem raised by the leaving of Blough (probably initially on a one year leave of absence) and Brownlee. Schultz suggested that the Department had two alternatives open to it: a temporary replacement (construed broadly) and a permanent appointment of a top ranking person.

The Department considered first possible candidates for permanent appointment. Attention centered on George Stigler, Abba Lerner, Kenneth Boulding, Leonid Hurwicz, Kenneth Arrow, and Lawrence Klein. For a temporary appointment Schultz suggested Gunnar Myrdal.

[Meeting began at 3:30 pm and ended 5:45 p.m.]

_________________________

From the MINUTES, Meeting of the Department,
May 30, 1950.

Present: T. W. Schultz, R. Burns, D. G. Johnson, E. J. Hamilton, F. H. Knight, L. Metzler, R. Blough, F. H. Harbinson, A. Rees, H. G. Lewis, T. Koopmans, J. Marschak, M. Friedman.

Appointments

The discussion of appointments continued from the previous meeting. Schultz expressed the conviction that the time was propitious for a new permanent appointment. On Metzler’s suggestion, the Department returned to discussion of the following candidates for a permanent appointment: Stigler, Hurwicz, Boulding, Klein, Lerner, Arrow.

Several members of the Department stated that none of these men had all of the qualities sought: a good mind reaching out fruitfully in new directions in economics. It was agreed, however, that there were no likely candidates possessing these qualities in a high degree.

The chairman then polled those present with respect to their first choice (or ties for first) for a permanent appointment. As a result of the poll the list of candidates was narrowed to Hurwicz, Stigler, and Lerner. The chairman then polled those present on their position toward permanent appointment of each of these men.

The poll showed that of those present

4 would favor and 5 oppose the permanent appointment of Hurwicz
4 would favor and 5 oppose the permanent appointment of Lerner
6 would favor and 6 oppose the permanent appointment of Stigler

A motion was passed instructing the chairman to poll the absent members of the Department in the same way on the appointment of Hurwicz, Lerner, and Stigler and to report back to the Department for further discussion.

[Meeting began at 3:30 pm and ended 6:15 p.m.]

_________________________

From the MINUTES, Meeting of the Department,
June 8, 1950.

Present: T. W. Schultz, H. G. Lewis, D. G. Johnson, J. Marschak, H. Kyrk, P. Thomson, M. Friedman, T. Koopmans, A. Rees, E. J. Hamilton, F. H. Knight, R. Blough.

Appointments

Schultz reported that he had polled Kyrk, Thomson, Mints, and Nef (but had not heard from Goode) on the matter of a permanent appointment for Stigler or Hurwicz or Lerner. The upshot of the poll was that the Department, the Chairman not voting, was evidently divided in its rating of Stigler for a permanent appointment; both permanent members and temporary members of the faculty showed an even division. The Chairman explained that he would abstain from voting on the belief that the Department was not now prepared to advance, with a strong meeting of minds, a strong case to the Central Administration for a permanent appointment. Schultz proposed that we investigate a slate of names for a one-year appointment.

A motion was passed authorizing the Chairman to put Gunnar Myrdal in the first position on the slate for a one-year appointment.

Successive motions passed by the Department added the following names to the slate:

Nicholas Kaldor   Simon Kuznets
Arthur F. Burns
H. M. Henderson
W. Vickrey
A. Hart
H. Stein

The Department then, following the system of ranking used in fellowship appointments, ranked these seven persons. The rank order follows:

1. Kaldor
2. Burns
3. Henderson
4. Kuznets
5½. Vickrey
5½. Hart
7. Stein

[Meeting began at 3:30 pm and ended 6:00 p.m.]

Source: University of Chicago Archives, Department of Economics Records, Box 41, Folder 12.

Image Source: Social Science Research Building.  University of Chicago Photographic Archive, apf2-07466, Special Collections Research Center, University of Chicago Library.

 

Categories
Harvard Seminar Speakers

Harvard. International Economic Relations Seminar. Haberler and Harris, 1940-45

 

The most famous economics seminar at Harvard University in the history of economics is undoubtedly the fiscal policy seminar run by John Williams and Alvin Hansen. A list of that seminar’s speakers and their topics was included in an earlier post. Below I provide the reported speaker’s and topics for the “younger” international economic relations seminar jointly organized by Gottfried Haberler and Seymour Harris during the War years.

___________________________________

EXPANSION OF THE SEMINAR PROGRAM

Several additions have been made in the seminar program of the School [of Public Administration] for the year 1940-1941. Professors Haberler and Harris are presenting a seminar on international economic relations. We planned our seminar program in 1937 on the assumption that it was wise to begin with domestic problems despite the fact that a number of the Faculty had special interests in the international field. In view of the events of the last few years, it seems highly important to develop these interests. The seminar given by Professors Haberler and Harris deals with the application of the principles of international trade to current problems…

Source: Harvard University. Report of the President of Harvard College and Reports of Departments for 1939-40, p. 306.

___________________________________

1940-41
INTERNATIONAL ECONOMIC RELATIONS SEMINAR
[partial list]

[Seven of the meetings of the Fiscal Policy Seminar were held jointly with other seminars – four with the International Economic Relations Seminar and three with the Agricultural, Forestry, and Land Policy Seminar.]

 

October 11. SVEND LAURSEN, Student, Graduate School of Arts and Sciences, Harvard University.

Subject: International Trade and the Multiplier. (Joint meeting with Fiscal Policy Seminar.)

February 21. HARRY D. WHITE, Director, Division of Monetary Research, United States Treasury Department.

Subject: Blocked Balances. (Joint meeting with Fiscal Policy Seminar.)

March 21. RICHARD V. GILBERT, National Defense Advisory Commission.

Subject: The American Defense Program. (Joint meeting with Fiscal Policy Seminar.)

May 2. GUSTAV STOLPER, Financial Adviser.

Subject: Financing the American Defense Program. (Joint meeting with Fiscal Policy Seminar.)

 

Source: Harvard University. Report of the President of Harvard College and Reports of Departments for 1940-41, p. 323 ff.

___________________________________

INTERNATIONAL ECONOMIC RELATIONS SEMINAR:
1941-1942. Professor Haberler and Associate Professor Harris

In 1941-42 the seminar devoted its attention to war and post-war problems in the field of International Economic Relations. A few meetings were spent on the discussion of fundamental theoretical problems. During the first semester all meetings were taken up by papers of outside consultants and their discussion. In the second semester student reports were presented and discussed, and a few extra meetings were arranged for outside speakers. The consultants and their topics were as follows:

 

October 1. EUGENE STALEY, Fletcher School of Law and Diplomacy. Economic Warfare.

October 8.[**] CHARLES P. KINDLEBERGER, Federal Reserve Board. Canadian-American Economic Relations in the War and Post-War Period.

October 15.[**] A. F. W. PLUMPTRE, University of Toronto. International Economic Position of Canada in the Present Emergency.

October 22. HEINRICH HEUSER, Fletcher School of Law and Diplomacy. Exchange Control.

October 29. FRITZ MACHLUP, University of Buffalo. The Foreign Trade Multiplier.

November 5. HENRY CHALMERS, United States Department of Commerce. Trade Restrictions in Wartime.

November 12. ARTHUR R. UPGREN, United States Department of Commerce. International Economic Interest of the United States and the Post-War Situation.

November 19. OSKAR MORGENSTERN, Princeton University. International Aspects of the Business Cycle.

November 28.[*] NOEL F. HALL, British Embassy. Economic Warfare.

December 5.[*] ROBERT BRYCE, Department of Finance, Canada. International Economic Relations with Special Reference to the Post-War Situation.

January 26.[*] PER JACOBSSEN, Bank for International Settlements. The Problem of Post-War Reconstruction.

February 13.[*] JACOB VINER, University of Chicago. Monopolistic Trading and International Relations.

February 18. H. D. FONG, Director, Nankai Institute of Economics, Chungking, China. Industrialization of China.

February 25. MICHAEL HEILPERIN, Hamilton College. International Aspects of the Present and Future Economic Situation.

March 11. JACOB MARSCHAK, New School for Social Research. The Theory of International Disequilibria.

March 14.[*] RICHARD M. BISSELL, JR., Yale University and the United States Department of Commerce. Post-War Domestic and International Investment.

March 18. ANTONIN BASCH, Brown University. International Economic Problems of Central and Southeastern Europe.

March 20.[*] ALBERT G. HART, University of Iowa. The Present Fiscal Situation.

April 10. ABBA P. LERNER, University of Kansas City. Post-War Problems.

May 8. HORST MENDERSHAUSEN, Bennington College. International Trade and Trade Policy in the Post-War Period.

 

Six of these were joint meetings with the Fiscal Policy Seminar [*] and two were joint meetings with the Government Control of Industry Seminar[**].

Student reports were presented on the following subjects:

Argentine International Trade.
Exchange Control in Argentina.
Some Aspects of Sino-Japanese Trade.
International Effects of Price Ceilings.
Location Theory and the Reconstruction of World Trade.
Some Post-War Politico-Economic Problems of the Western Hemisphere.
Economic Problems and Possibilities of a Pan Europe, Pan America and Similar Schemes.
The Balance of Payments of China.

 

Source: Harvard University. Report of the President of Harvard College and Reports of Departments for 1941-42, pp. 344-346.

___________________________________

INTERNATIONAL ECONOMIC RELATIONS SEMINAR
1942-43. Professor Haberler

A larger portion of the time of the seminar than usual was devoted to the discussion of fundamental principles of international trade and finance. This was due to the fact that the graduate course on international trade (Economics 143) was not offered, and the seminar had to take over to some extent the functions of the graduate course.

There were eleven meetings with outside consultants, of which eight were joint meetings with the Fiscal Policy seminar. The smaller number of students made it advisable to combine the two seminars more frequently than usual. The consultants and the topics discussed with them were as follows:

 

November 13. Professor FRITZ MACHLUP, University of Buffalo. (Joint meeting with Fiscal Policy seminar.)

Subject: National Income, Employment and International Relations; the Foreign Multiplier.

November 18. Dr. THEODORE KREPS, Economic Adviser, Board of Economic Warfare, Office of Imports.

Subject: Some Problems of Economic Warfare.

November 27. Hon. GRAHAM F. TOWERS, Governor, Bank of Canada. (Joint meeting with Fiscal Policy seminar.)

Subject: Canadian War Economic Measures.

December 4. LYNN R. EDMINSTER, Vice-Chairman, U. S. Tariff Commission. (Joint meeting with Fiscal Policy seminar.)

Subject: Post-War Reconstruction of International Trade.

December 11. Professor SEYMOUR E. HARRIS, Director, Office of Export-Import Price Control, Office of Price Administration. (Joint meeting with Fiscal Policy seminar.)

Subject: Trade Policy in Wartimes.

February 12. THOMAS MCKITTRICK, President, Bank for International Settlements. (Joint meeting with Fiscal Policy seminar.)

Subject: The Bank for International Settlements.

February 24. Dr. LEO PASVOLSKY, State Department. (Joint meeting with Fiscal Policy seminar.)

Subject: Post-War Problems in International Trade.

March 3. P. T. ELLSWORTH, War Trade Staff, Board of Economic Warfare.

Subject: The Administration of Export Control.

April 12. EMILE DESPRES, Office of Strategic Services, Washington, D. C. (Joint meeting with Fiscal Policy seminar.)

Subject: The Transfer Problem and the Over-Saving Problem in the Pre-War and Post-War Worlds.

April 16. Dr. ALBERT HAHN. (Joint meeting with Fiscal Policy seminar.)

Subject: Planned or Adjusted Post-War Economy.

April 20. Dr. ALEXANDER LOVEDAY, League of Nations.

Subject: European Post-War Reconstruction.

 

Student reports were presented on the following subjects among others: practice and theory of an international bank; post-war industrialization of China; coordination of fiscal policy in different countries; international position of the Brazilian economy; international commodity agreements; international implications for fiscal policy; British exchange equalization account; and Argentine exchange control.

Twelve students were enrolled in the seminar of which four were Littauer fellows, seven graduate students from the Graduate School of Arts and Sciences, and one from the College.

 

Source: Harvard University. Report of the President of Harvard College and Reports of Departments for 1942-43, pp. 246-247.

 

___________________________________

INTERNATIONAL ECONOMIC RELATIONS SEMINAR
1943-44. Associate Professor Harris

A new approach was tried in the International Economic Relations Seminar this year. We paid particular attention to the international economic problems of Latin America and especially to the problems raised by the great demand for Latin American products for war, the expansion of exports and of money, and the resulting inflation. Attention was also given to the transitional problems in the postwar period, particularly to the adjustments that will be required in exports, imports, capital movements, exchange rates, and the allocation of economic factors. In the course of the year leading government authorities on Latin American economic problems were invited to address meetings of the seminar, which were frequently joint meetings with the Fiscal Policy Seminar or the students of the graduate course in international organization.

The schedule of meetings for 1943-44 was as follows:

 

November 12. Professor HARRIS.

Subject: Inflation in Latin America.

December 9. Dr. CORWIN EDWARDS, Chairman, Policy Board of the Anti-Trust Division of the Department of Justice and Chief of Staff of the Presidential Cooke Commission to Brazil.

Subject: Brazilian Economy.

December 17. Dr. HARRY WHITE, Director of Monetary Research, Treasury Department.

Subject: Problems of International Monetary Stabilization.

January 6. Professor HARRIS.

Subject: International Economic Problems of the War and Postwar Period.

January 10. Professor HABERLER.

Subject: Reparations.

January 14. Dr. N. NESS, Member, Mexican-U. S. Economic Commission.

Subject: Mexico.

January 17. Dr. BEARDSLEY RUML, Chairman, Federal Reserve Bank of New York.

Subject: Economic Budget and Fiscal Budget.

January 21. Dr. P. T. ELLSWORTH, Economic Studies Division, Department of State.

Subject: Chile.

January 24. Dr. DON HUMPHREY, Special Advisor on Price Control to Haitian Government; Chief, Price Section, O.P.A.

Subject: Haiti.

January 31. Dr. ROBERT TRIFFIN, Member, U. S. Economic Commission to Paraguay.

Subject: Money, Banking, and Foreign Exchanges in Latin America.

February 4. Dr. MIRON BURGIN, Office of Coordinator of Inter-American Affairs.

Subject: Argentina.

February 9. Dr. FRANK WARING, Director, Research Division, Office of Coordinator of Inter-American Affairs.

Subject: Broad Aspects of Latin-American Economics.

February 10. Dr. BEN LEWIS, Head of Price Control Mission to Colombia, Special Assistant to the Price Administrator.

Subject: Colombia.

March 9. Dr. HENRY CHALMERS, Department of Commerce.

Subject: Inter-American Trade Practices.

March 31. Mr. HENRY WALLICH.

Subject: Fiscal Policy and International Equilibrium.

 

Source: Harvard University. Report of the President of Harvard College and Reports of Departments for 1943-44, pp. 271-2.

___________________________________

INTERNATIONAL ECONOMIC RELATIONS SEMINAR
Professor Haberler and Associate Professor Harris

The seminar meetings in the year 1944-1945 may be arranged under the following headings:

  1. Exchanges, Controls, and International Trade (8 meetings)
  2. Regional Problems (8 meetings).
  3. Regional and International Aspects of Domestic Problems (8 meetings).
  4. Lectures and Discussions on International Trade by Professors Haberler and Harris (8 meetings).

Four of the papers presented at these meetings were subsequently published in economic journals.

The schedule of meetings for 1944-1945 was as follows:

November 16. Dr. RANDALL HINSHAW, Federal Reserve Board.

Subject: American Prosperity and the British Balance-of-Payments Problem. (Published in the Review of Economic Statistics, February 1945.)

December 11. EDWARD M. BERNSTEIN, Assistant Director, Division of Monetary Research, Treasury Department.

Subject: The Scarcity of Dollars. (Published in The Journal of Political Economy, March 1945.)

December 15. Dr. FRANCIS MCINTYRE, Representative of the Foreign Economic Exchange on Requirements Board of the War Production Board.

Subject: International Distribution of Supplies in Wartime.

December 21. Dr. ALEXANDER GERSCHENKRON, Federal Reserve Board.

Subject: Some Problems of the Economic Collaboration with Russia.

January 11. Dr. WOLFGANG STOLPER, Swarthmore College.

Subject: British Balance-of-Payments Problem After World War I.

January 22. Dr. WALTER GARDNER, Federal Reserve Board.

Subject: Some Aspects of the Bretton Woods Program.

January 26. Dr. WILLIAM FELLNER, University of California.

Subject: Types of Expansionary Policies and the Rate of Interest.

January 29. Professor WALTER F. BOGNER, Dr. CHARLES R. CHERINGTON, Professors CARL J. FRIEDRICH, SEYMOUR E. HARRIS, TALCOTT PARSONS, ALFRED D. SIMPSON, and Mr. GEORGE B. WALKER.

Subject: The Boston Urban Development Plan.

March 5. Dr. ROBERT TRIFFIN, Federal Reserve Board.

Subject: International Economic Problems of South America.

March 19. Dr. LOUIS RASMINSKY, Foreign Exchange Control Board, Ottawa, Canada.

Subject: British-American Trade Problems from the Canadian Point of View. (Published in the British Economic Journal, September I945.)

March 22. Dr. ROBERT A. GORDON, War Production Board.

Subject: International Raw Materials Control: War and Postwar.

March 26. Dr. HERBERT FURTH, Federal Reserve Board.

Subject: Monetary and Financial Problems in the Liberated Countries.

April 2. Dr. LLOYD METZLER, Federal Reserve Board.

Subject: Postwar Economic Policies of the United Kingdom. (An article based on this paper and written in collaboration with Dr. RANDALL HINSHAW was published in The Review of Economic Statistics, November 1945.)

April 16. Professor EDWARD S. MASON, State Department, Washington.

Subject: Commodity Agreements.

April 23. Dr. ABBA P. LERNER, New School for Social Research, N. Y.

Subject: Postwar Policies.

April 27. Professor JOHN VAN SICKLE, Vanderbilt University.

Subject: Wages and Employment: A Regional Approach.

May 14. Dr. E. M. H. LLOYD, United Relief and Rehabilitation Administration, British Treasury.

Subject: Inflation in Europe.

May 28. Professor LEON DUPRIEZ, University of Louvain, Belgium.

Subject: Problem of Full Employment in View of Recent European Experience.

May 29. Professor SEYMOUR E. HARRIS, Professor WASSILY W. LEONTIEF, Professor GOTTFRIED HABERLER, Professor ALVIN H. HANSEN.

Subject: The Shorter Work Week and Full Employment.

 

Source:   Harvard University. Report of the President of Harvard College and Reports of Departments for 1944-45, pp. 285-6.

 

Categories
Curriculum Economics Programs Yale

Graduate Training in Economics. Report of Panel Discussions at Yale. 1956

 

 

 

During the fall and early winter of 1954-55, Richard Ruggles and colleagues in the Yale economics department organized a series of interviews with representatives of business, government, international organizations, and universities to review the ultimate goals of a graduate education in economics and to identify future desirable directions the evolution of economics training might take. The interviews were followed by panel discussions in the Spring of 1955 attended by, among others, seven future economics Nobel prize winners. Today’s posting is a transcription of the final report printed in 1956. 

I came across a preliminary draft of the report in the Milton Friedman papers at the Hoover Institution Archives filed among his correspondence with Richard Ruggles and wondered whatever happened to the project. The report was never really published and survives as part of the “pamphlet literature”.  Only recently did I find a printed copy of the final report in John Kenneth Galbraith’s papers in the John F. Kennedy Presidential Library. The relative obscurity of this report can perhaps be attributed to its “Smoothie” style that has managed to blend panel member ideas and opinions into mere minutes of discussions sans quote or illustration. The report’s temporal proximity to the 1953 Bowen report (Graduate Education in Economics, AER, September 1953) could have left journal editors cold as well.

Since the primary goal of Economics in the Rear-view Mirror is to assemble artifacts to help us follow the historical development of the education of economists in the United States, the Ruggles Report of 1956 is worth rescuing from its undeserved obscurity in archival vaults.

________________________________

 

[1]

GRADUATE TRAINING IN ECONOMICS
A Report on Panel Discussions at Yale
YALE UNIVERSITY
1956

 

[2]

A restudy of graduate education in economics has recently been undertaken at Yale, with the aid of a grant from the Ford Foundation. This study involved two steps. First, economists in universities, government, and business were interviewed to determine what they thought the major problems in training economists were at present. These views were summarized in the form of an agenda, which was then discussed by five panels of economists. This report presents the views of the panel members, as developed in these discussion groups.

The following people participated in the panel discussion and in the revisions of the report.

Panel members:

Robert Adams, Standard Oil Company of New Jersey
Sydney Alexander, Massachusetts Institute of Technology
Kenneth Arrow, Stanford University
G.L. Bach, Carnegie Institute of Technology
William Baumol, Princeton University
E. G. Bennion, Standard Oil Company of New Jersey
Henry Bloch, United Nations
Howard Bowen, Grinnell College
Sune Carlson, United Nations
Gerhard Colm, National Planning Association
Ross Eckler, Bureau of the Census
Solomon Fabricant, national Bureau of Economic Research
Milton Friedman, University of Chicago
Albert Hart, Columbia University
Leonid Hurwicz, University of Minnesota
Dexter Keezer, McGraw-Hill Publishing Co.
Simon Kuznets, Johns Hopkins University
Stanley Lebergott, Bureau of the Budget
Wassily Leontief, Harvard University
Ben W. Lewis, Oberlin College
John Lintner, Harvard Business School
Edward S. Mason, Harvard University
James Nelson, Amherst College
Donald Riley, Bureau of the Budget
Paul Samuelson, Massachusetts Institute of Technology
Robert Strotz, Northwestern University
Clair Wilcox, Swarthmore College

 

Yale committee:

Richard Ruggles, Chairman
Wight Bakke
William Fellner
Kent Healy
John Miller
John Sawyer
James Tobin
Robert Triffin

 

[3]

The Role of Graduate Education in Economics

THE OBJECTIVES OF GRADUATE EDUCATION IN ECONOMICS which were most frequently mentioned by the panel members were (1) to develop economists who can push back the frontiers of economics; (2) to prepare economists for teaching, not only at the undergraduate level but also in graduate economics departments and business schools; (3) to train individuals who are capable of carrying out research for business, government, labor, and other research organizations; (4) to develop economists who can serve in policy guidance positions in business, government, and labor unions. The panel members agreed that the curriculum of graduate education in economics can no longer be organized exclusively about scholars; it has become essential to produce economists who can do, not just know. Primary emphasis in the past has been placed upon the production of teachers, and although this is an important function, focusing on it may develop a more restricted concept of education than is appropriate today.

The frontier of economic knowledge.

The continual emergence of economists who are capable of contributing to the substance of economics is essential for the vitality of the field. Of course, every student who goes through a graduate school should not be expected to make such a contribution; many are needed to practice the art and science of economics for more immediate objectives in teaching, in applied economics in business and government, and in less basic research in the academic world, business, and government. Nevertheless, the graduate school program should be such as to encourage research of a basic nature and to acquaint students with it. Only by such investment can economics be expected to develop. Such an orientation is useful also for those who do not go on to make substantial new contributions. It provides a [4] necessary perspective as to the current status of economic knowledge and the bases on which it resets, and points up gaps in economic knowledge and the process by which the evolution of economic thought comes about. Accent on the encouragement of basic research should not be construed, however, as implying that large amounts of learning and scholarship should be the aim. Rather it implies that the creative talents of the individual should be stimulated, and that the individual be trained in the necessary tools to do such research. These aims are complementary to the other objectives of graduate training, not competitive with them.

Research training for business and government.

In recent years, there has been an increasing use of economists for research purposes in business and government. Projections of future demand, analyses of the impact of various market forces, problems of taxation and government expenditure, analyses of productivity changes, studies of business fluctuations, and various international problems related to trade and foreign economic policy all have required that a considerable amount of economic research be carried out. Graduate schools have not generally taken specific cognizance of the needs of these groups so that new Ph.D.’s going into these areas often require a considerable training period before they become useful to their organizations. When the organization does not have available senior staff capable of carrying out such training on the job, the result is that lower grade work is turned out. It is recognized, of course, that schooling cannot entirely substitute for experience, and that some training on the job will always be necessary, but the question still remains whether the present graduate school training is as appropriate as it might be for meeting the research needs of business and government.

Policy and administrative guidance in business, government, and labor.

Besides the technical research uses of economists in business, government, and labor, economists are needed in a more operating [5] capacity, where day-to-day decisions and advice are required without any formalized research work. Advisors are required at the policy level in large corporations. Banks, insurance companies, large manufacturing firms, and labor unions are employing more and more people in this capacity. Government and international organizations need trained economists to serve as administrators of various programs. These needs are growing in importance as the complexities of economic life increase. Again, most graduate schools have not been particularly attuned to meeting this sort of need.

Teaching.

To a very large degree, teaching is a derivative of the other purposes of economic training. Teachers should be expected to be able to teach those things which are useful in the training of economists. Thus, at the graduate level the objectives outlined above would be pertinent; teachers should be trained to meet these objectives. The problem of undergraduate teaching of economics may at first appear to pose somewhat different requirements, but closer examination indicates that its objectives should be closely allied with the objectives cited above, lest it become too academic and unrelated to the current practice of economics. Undergraduate teachers need to be trained broadly and to have a good general perspective about economics. The development of teachers who are interested in the furthering of economics as a science is necessary in order to prevent the teaching of economics from becoming a sterile academic exercise. The crucial question here is the ability to teach effectively, and to keep on doing it through time—to keep alive, stimulated and stimulating.

 

[6]

Requirements Posed by the Objectives of Graduate Training in Economics

THE OBJECTIVES OF GRADUATE TRAINING IN ECONOMICS are largely complementary in the requirements they pose; there seems little ground for suggesting that individuals expecting to go into different areas of economics should have greatly different and unrelated programs. It was thought that the basic requirements common to all the objectives could be classified into four major categories: (1) a common core of economic knowledge; (2) the ability to present ideas coherently; (3) the ability to do research; and (4) the specialized training in the area of the student’s greatest interest.

No strong line of distinction can in fact be drawn between knowledge, on the one hand, and the ability to present ideas coherently and the ability to do research, on the other hand. A person who does not have the ability to express ideas coherently or the ability to do research cannot be said to possess knowledge of his subject. True knowledge is more than the capacity for parrot-like repetition of what this, that, or the other economist said, and what this, that, or the other formula is, and unless research is narrowly defined as the analysis of empirical data of a limited kind, really operative knowledge is included under either the ability to present ideas coherently or the ability to do research or both. Thus, the teaching involved in imparting the common core of knowledge (as well as that involved in specialized training) should be such as to produce in the student clarity of thinking which should make clear writing a necessary consequence; and, also, the teaching involved in imparting the common core of knowledge (and specialized training) should be such as to leave the student with a clear idea of what research means, and how the interplay of hypotheses with tests based on empirical data results in acceptable knowledge.

In spite of the obvious interrelationship of the four major [7] categories listed above, however, it will be useful to consider them one at a time.

 

COMMON CORE OF ECONOMIC KNOWLEDGE

All economists should have a general acquaintance with the basic ideas in economics, and all should be equipped with the tools and the general empirical knowledge about modern economic systems that will provide a basis for economic research, policy guidance, and teaching. The common core consists of (1) a set of analytical tools, (2) a way of handling the tools in research and problem solving, and (3) certain institutional knowledge about the economic world. This common core is necessary not only to meet the above objectives, but also so that economists will be able to communicate with each other, and so that mobility among different uses of economists will be preserved. The substance of economics itself will be enriched if individuals can move freely from one area to another. For example, it is beneficial for the development of the profession if economists can move between business and government, on the one hand, and teaching, on the other. Similarly, research individuals should have the same sort of general background as those who are faced with administrative problems. The existence of a common core helps to ensure this, and is some protection against excessive compartmentalization and overspecialization in the profession. The problem of core training is one of balancing the desirability of having a number of essential requirements included in each student’s program with that of having the minimum amount of formal requirements.

With respect to the nature of the common core, there was fairly general agreement among those participating in the panels, and the conclusions reached are not strikingly different from the current practice in many graduate schools or the objectives expressed in the Bowen Report. There was a general feeling that some reorientation and redesign within the accepted framework might be in order, but that the general framework itself [8] need not be significantly altered. The content envisaged would include economic theory, economic history, mathematics, and statistics.

Economic theory.

The theory requirement in the common core should probably be the most intensive of all the requirements. At least one and probably two full years of formal classwork in economic theory were considered necessary, supplemented by outside reading to fill in gaps not taken up in the formal courses. The courses themselves would not be entirely devoted to a formal presentation of certain specialized areas of theory, but should give students the ability to use theory effectively in handling problems. The work should cover modern theory in most areas of economics, and it should also be tied in with both the history of economic thought in these areas and some of the historical and institutional background that provides the context for the theory.

Economic history.

Economic history as a core component should be distinguished from economic history as a special field. The purpose of the economic history requirement should be one of literacy, to insure that the student has some perspective with respect to how economics is related to various aspects of human development. This requirement can provide the thread of continuity and integration which is normal lacking from work at graduate level. The growth and development of economic institutions in the various specialized areas should be treated in relation to each other, together with the relation of social and political history to economic development and the role of geographic location as a determinant of economic development.

Mathematics.

The purpose of the mathematics requirement as a part of the core is partly to serve as a necessary tool for the mathematical economics and statistics, and partly for general literacy. It would [9] be desirable, of course, for students to have a proper mathematical background when they enter graduate school. Unfortunately, such a requirement is not easily enforced at this time, and it will generally be necessary for this deficiency to be made up either while a student is taking other work in graduate school or during the summers. In view of the specialized nature of the mathematics required for economists, it may well be that a specialized course drawn up specifically for economists or for social scientists would be the most efficient way to meet the need. Such a course would not be intended as a shortcut, but rather would attempt to give the student those areas of mathematics which are relevant to social science and to relate them to problems in economic theory, game theory, statistics, and econometrics. Literacy in the area of mathematics is important so that students will not be frightened by economics which is cast in symbolic terms. If there is to be communication among members of the profession, it is essential that all economists should have enough mathematics so that they can tell in a general way what articles in a mathematical form are about. This does not mean that those students who are not mathematically inclined should be forced to achieve mathematical fluency. However, all students should at least be required to have some minimum competence in mathematics.

Statistics.

As in the case of mathematics, statistics is partly a tool requirement and partly a literacy requirement. As a tool, students should be able to employ statistics for economic research. The traditional topics such as probability theory, statistical tests, and index numbers would all be covered. In addition, however, the student should learn how to handle basic empirical material in a systematic and orderly manner. The uses of accounting data, together with the meaning of various accounting classifications and accounting methods, should be studied. The student should also have a general knowledge of the sources of economic data, such as the kind of material contained in the various censuses of [10] the U.S., the national income statistics, and the types of economic information provided by the other agencies in the government. They should be familiar with the empirical work provided by non-governmental research institutions such as the National Bureau, and by international organizations. All of these are useful research tools, and they are also required for literacy in this area, so that the student will be able to appraise and evaluate empirical research.

Interdisciplinary training as related to the core of economics.

Considerable attention has been focused recently upon the desirability of having students know about fields other than economics, so that useful cross-fertilization can take place among the disciplines, and so that economics can be used more effectively in helping to handle public and private policy problems. It is argued that training in other disciplines will give the student greater breadth and make his economics training more meaningful. There was a general consensus among the members of the panels, however, that elementary survey courses in other disciplines would be of limited usefulness, and would expand the common core to a point where it would seriously infringe upon the freedom of students to follow lines of their own interest. Undergraduate training supposedly gives a student breadth; if it has failed in doing this the lack should be recognized as a gap in the student’s training. It is questionable, however, whether a graduate school should take formal cognizance of such gaps, as it does in the case of mathematics, and make provision in the graduate school curriculum for filling them. Where the gaps are extremely serious, the student should probably be encouraged to attend summer school, an/or do special reading, to make up the deficiencies. But it does not seem that the subject matter of interdisciplinary training and the deficiencies of preparation in the students are sufficiently clearly defined to make courses in them practical. Experiments might usefully be tried in this area, but they should be regarded strictly as experiments, [11] which might eventually yield elements that should be incorporated into the common core.

The extent and timing of the common core.

In terms of formal requirements, the common core should probably not exceed four or five year courses, depending upon whether or not the student can anticipate the mathematics requirement. In addition to this formal work, however, it might be desirable to provide for some sort of tutorial instruction to fill in gaps not covered in the courses and to follow up lines of special interest to the individual student. Such tutorial instruction would provide an element of flexibility not obtainable in formal classwork. With respect to timing, it seems logical that the major portion of the core would be covered in the first year, inasmuch as it provides tools used at later stages in graduate work. On the other hand, some time should be left in the first year for students to take courses of their own selection. Students should have an opportunity to sample several specialized areas before finally determining the area in which they are most interested.

The Ability to Express Ideas Coherently

The economist should have the ability to express his ideas coherently, and to move easily between the abstractions posed by economic analysis and the empirical elements of the problems with which he deals. This requirement is more than that of being able to write grammatical English; it involves training in the organization of ideas and the development of perspective. Rigor and clarity is essential if the profession is to serve its many potential functions. One of the major complaints of people who hire economists in business and government is that the products of graduate schools whom they hire do not have this ability to present their ideas coherently. They often express the opinion that economists who are intending to go into business and government should receive special training in this respect. However, [12] it is not any less important that individuals going into pure research or teaching should be trained to express their ideas coherently. Perhaps the reason teaching and academic research have not appeared to suffer as much in this respect lies in the lack of direct supervision of such individuals by supervisors who bear the responsibility for their written and oral presentations.

As already indicated, the ability to express ideas coherently is not merely a problem of correct grammar, but rather involves the organization of ideas in a meaningful manner. Unless a student can express an idea clearly, he does not really understand it. Thus, the ability to express ideas coherently is highly related to the problem of substance, and is properly the responsibility of a graduate school. Some students have difficulty in writing because they have little or nothing to say. They have not developed habits of creative thinking, and do not know how to approach a subject.

Because the economist usually crystallizes the results of his work in written form the writing itself is a tool, and is part of the basic methodology of the profession. In other disciplines such methodological tools are given explicit consideration. For example, in the sciences, students are thoroughly trained in laboratory work. In mathematics, students are drilled in working through problems. In law, briefs and case studies are written. In medicine, the internship trains the student in the handling of actual medical cases. Few graduate schools of economics, however, have considered writing explicitly as a tool of the profession, and therefore relatively little accent has been placed upon training the student adequately in this function.

The Ph.D. thesis, traditionally the masterpiece of a student being trained for the doctorate, does not fulfill this need. All too often it is instead a traumatic experience which leaves the student scarred but untrained. In a great many instances, furthermore, the thesis is done by the student out of residence, and the supervision of the writing of it leaves much to be desired. The student often attempts to write the thesis while he is pursuing another job on a full-time basis, and the writing may take [13] a period of five or six years. The hurdle is so great, as a matter of fact, that a large proportion of students who have completed everything but the thesis never finish it. Also, the moral pressure on professors to approve theses of students who have spent a large number of years on them is very great, with the result that the thesis itself need only show effort and length to be acceptable. In other words, the Ph.D. thesis is quite unsatisfactory for teaching students how to write, and because of the institutional considerations involved this failure cannot be corrected merely by exhorting students and teachers to greater effort and higher standards.

The members of the panels believed that the solution to the problem of training students to write coherently lies in the direction of more writing practice early in the graduate training program, and reliance on a larger number of shorter papers (5 to 10 pages) rather than a small number of major papers. This process should intimidate the student less, offer him more practice in organizing material, and make the task of criticizing and evaluating any given paper simpler.

One important aspect of training students to write, now largely neglected, is provision for revising and reworking papers. So much effort goes into the original writing of a lengthy paper, and the task of reworking it is so great, that most of the student’s writing tends to be a single-shot experience. In many cases the student never even seriously re-reads what he has written after he finishes it. In order to promote the reading and criticism of papers, it was suggested that some of the papers be duplicated and discussed in essay seminars attended by both students and faculty. Students should learn from such a procedure not only when their own work is presented but also from the problems encountered by other students. In this connection also, all papers need not be written in the confines of formal courses. The tutorial function spoken of in the previous section might well bear some of the brunt of criticizing short papers.

Courses involving group research would provide an opportunity for students to prepare papers in conjunction with each [14] other. Such joint papers would force the students to discuss the organization and presentation of the material, so that an agreed-upon version may be arrived at. This practice will prepare students for the sort of writing experience they are likely to encounter in business, government, or other group research.

If the writing of papers is to be stressed as a part of the graduate training program, it is only proper that it should assume a more significant role in the grading system. The student who can produce a first-class report at this own leisure, using the materials freely available to him, may well be a better economist than one who is more facile in showing his learning well in an examination but who may also be less proficient in turning out an independent piece of research. Present grading systems rely heavily upon examinations, which may test the student’s leaning ability but do not ordinarily test his ability to produce a well-conceived and well-executed report. The comprehensive examinations weigh very heavily in determining whether students are permitted to proceed and what kind of financial aid they are given. At both the course level and at the comprehensive examination level, it would be possible to give greater weight to written reports in the grading scheme. For the comprehensive examination, the student might be required to present what he considered the best two or three papers he had written. An evaluation of these papers would add a significant new dimension to the judgment of the abilities of students at this stage. By giving reports and papers a significant weight in the grading structure of the graduate school, students would be encouraged to revise and rework their manuscripts to a greater extent than they now do. Originality would be rewarded just as learning ability is now rewarded.

Research Competence

Because so many economists are required to do research of some sort in their work, and because all economists must be able to analyze and evaluate the results of such research, research [15] training is essential. The tools of economic research are, of course, necessary at least in some degree, but fully as important as the teaching of tools is the actual training of students to do research by doing it. The student emerging from graduate school should be able to carry through a piece of research in a systematic and meaningful manner. Students must be trained to set out a problem, design their work program with reference to this problem, carry out the basic work utilizing pertinent sources and appropriate methods, and finally, evaluate the results of this research, relating them to the original problem and appraising their validity.

A number of members of the panels felt that economic research generally suffered from a lack of respect for discipline and rigor. Casual empiricism, rather than scientific testing of hypotheses, is all too frequent. In many major pieces of research the sources and methods behind the results are not indicated adequately. These faults, they believed, are the result of inadequate teaching of research methods.

The misapplication of research tools, or the failure to apply suitable tools, is also widespread in much current economic research. The research worker may carry extremely unreliable estimates out to a number of decimal places, causing an inordinate amount of computational effort and lending a spurious appearance of accuracy. At the same time, this same research worker may gloss over important characteristics of his material which should have been tested for bias or general inconsistency by the use of fairly ordinary and straightforward statistical testing procedures.

The lack of research competence is also evident in the formulation of research problems. Often the reader of a research paper is at a loss to discover just what is being undertaken, and whether it was in fact achieved. This confusion often stems from a lack of clarity on the part of the original research worker in the conception of his problem, even more than from his presentation of it. It is very important that those embarking upon research recognize the importance in the research process of the original [16] conception of the problem and the design of the research to fit the problem.

These faults in economic research, combined with indecisiveness on the part of the individual research worker, lead to a considerable amount of floundering and waste motion. It is frequently necessary to re-do a piece of research because the formulation of the problem was inadequate. The failure to apply the proper tools at the proper time in the research process also may require that much of the work be redone, to make adjustments the need for which becomes obvious at a later stage in the research process. The prevalent lack of discipline and rigor makes all these revisions of portions of the research process extremely difficult, so that in fact the work usually must be completely redone, very often with quite different results.

In the light of these difficulties, research training should start early in the student’s graduate career and continue throughout its duration. Although in his first year the student will not have the necessary background and tools to do very much economic research, even at this early date practice with simple research problems would be useful in acclimating students to the various problems that research poses earlier in their careers rather than later. More of the student’s time can then be focused at a later stage on problems of a more substantive nature. It is well known that the greater part of time now spent on the Ph.D. thesis is spent in floundering around trying to select a problem and decide just how to carry it out. More and earlier practice in research might avoid much of this floundering.

The assignment of a larger number of short research subjects seems generally preferable, at least in the earlier part of the graduate training, to concentration on a few more substantial topics. If a number of different subjects are assigned, the student is faced again and again with the problem of how to formulate the research objectives and how to design the research. A larger number of projects also will serve to introduce the student to a number of different areas of economics, rather than to concentrate his attention solely in one direction. The question of [17] whether specific research topics should be assigned or whether the student should be allowed to choose his own is not an easy one to answer. Probably some of each approach should be used. Assignment of topics has the advantage of training the students to write for a customer. Freedom of choice in topics, on the other hand, has the advantage of allowing students to follow areas of special interest—and also gives them practice in arriving at a decision.

One of the major objectives of research training should be practice in the handling of empirical material of all sorts. The student should become used to dealing with historical material, economic statistics from all kinds of sources, and also material from other disciplines. He should gain experience in the critical evaluation of definitions and concepts, and in the manipulation and recasting of material.

The form of research training should probably differ at different stages of the graduate training process. In the early stages it may well take the form of special workshop courses, together with some for the work done for tutorial purposes. At a later stage, internship in various research projects within the university might be advisable. If possible, summer internship programs with business, government, or economic research foundations would also be desirable. Finally, individual research relationships with the faculty members on the basis of research assistantships or apprenticeships would serve a valuable role.

The Ph.D. thesis should serve a major function in research training, and should provide a test of whether the student has achieved research competence. But the primary research training should be begun much earlier in the student’s career; it should not fall upon the thesis alone. The thesis may well emerge as an outgrowth of some earlier research project.

Specialization

Specialized training in specific fields is necessary so that economists can usefully bring to bear both the more detailed knowledge [18] of the institutions pertinent to the special area and the latest developments of economic analysis in this area. Without special field training, a student will not approach the frontier of any field, and will not have any training in depth. Specialized training, therefore, not only serves to equip a student to handle problems in a special area, but it also gives him training in depth as a background for understanding the process of research and appreciating the development of economics in general. In many special fields, economics alone will not be sufficient. Other disciplines are often required to enable the economist to deal with the specialized problems. In the area of corporate finance, law and accounting may be necessary. Law may also be necessary for public finance, labor, and international trade. Psychology or sociology may be pertinent to studies of consumer demand and labor. Each special field will necessarily entail the study of those portions of other disciplines which are germane to the set of problems encountered.

Under present circumstances specialization often tends to be somewhat superficial. The first year of graduate work is usually spent on the basic tool courses or general survey courses, and specialization is possible only during the second year of course work. A cumulative build-up of work within a special area is often impossible since the student finishes his term of residence at the end of the second year. Specialization may thus consist of one or two courses taken concurrently in the second year of graduate study.

The charge is often made that the areas of specialization offered tend to be too academic. Theory is extolled, and the actual work done by the student is largely confined to the library. Knowledge of the institutional setting of the special field tends to be slighted. There is little or no opportunity for internship in the special field during the period of graduate work.

Specialization may be conceived of as a highly detailed study of some small segment of economics or it may be conceived of as embracing a general area of problems for which other disciplines besides economics may also be relevant. Unfortunately, [19] present graduate training seems to emphasize only the first conception of specialization, but if the products of graduate schools are expected to serve as professionals in these areas the narrow concept of specialization must give way to the broader concept.

Finally, it is argued by representatives of both business and government that graduate training does not prepare students for the kind of work required in business and government. Unlike the conclusion in the previous sections with respect to the common core of economics, the ability to express ideas coherently, and the ability to do research, where it was concluded that the requirements are the same irrespective of whether the student wants to go into academic work, business, or government, additional training will depend upon the field the student decides to enter. The criticism that graduate schools at the present time do not offer appropriate specializations for students interested in business and government in the role of professional economists appears to be justified. The kinds of courses that would be required for such a specialization would cover such topics as projections, studies in demand and cost, and general economic accounting.

In order to correct the tendency toward superficiality, the student should customarily take two or three courses in a given special area, over a period of at least two years. This would provide the student with an opportunity to work in the area over a longer period, and so would permit a cumulative build-up.

Research work involving the handling of empirical material and/or field work should be undertaken simultaneously with the course work. Such research work might be part of an internship program, a workshop course, or an apprenticeship as a research assistant. In some cases, suitable summer employment might serve as part of the program.

As already indicated, training in related disciplines should accompany the work in the special field. Generally speaking, survey courses in related disciplines will not meet the need. Either courses especially designed to suit the area being studied or relatively advanced work within the other disciplines would be [20] appropriate in giving greater breadth to the program of specialization.

In order to meet the needs of business and government, a number of courses in fields not now generally offered could usefully be added. Such things as the problems of making projections, studies in cost and demand analysis, operations research, and economic accounting are all appropriate subjects, which could serve either as specialties in their own right or as valuable tool adjuncts in such fields as industrial organization, labor, and international trade.

The Role of the Ph.D. Thesis

In viewing the Ph.D. thesis as both a test of and a means of acquiring core knowledge, clarity of expression, and research competence, the panel members felt that the form of the thesis required some reconsideration.

The desirability of having the thesis written in residence is well recognized. Furthermore, the panel members generally agreed that it would seem sufficient as a requirement if students could turn out an article-length paper which would be of publishable quality. Such a short thesis could be examined and criticized in greater detail by the faculty, and, if needed, revised more often and more basically by the student. This does not mean that long Ph.D. theses should be prohibited; a student should have the right to undertake any task he wants to. Still, it does not seem unreasonable to require that even in the case of a long thesis the student shall, in order to meet the thesis requirement, present some piece of material not longer than 30 to 50 pages which can stand as an independent piece of writing, aside from possible appendices on sources and methods. Whatever he wants to do over and above this, of course, he can. It may well be argued that the short thesis should not be compulsory, but that it may be enough to announce to students that short theses are not only acceptable but encouraged. Several panel members felt that the short thesis might be inappropriate [21] for specific topics, and that the way should be left open so that the student could write a longer thesis if he chose to do so. There is danger in this approach, however, in that students may take the safe way out and write a long thesis much on the same basis that they write long answers to exam questions covering every possible facet of the question. In such a case the tendency to judge theses by the pound might continue.

If the requirement that the thesis be of publishable quality is seriously intended, it might be desirable to consider having the university undertake the actual publication, in the form of an annual series. If the theses are in fact held to a length of 30 to 50 pages, the cost of publishing them would not be excessive. Such an arrangement would have several advantages. First, it would tend to make the students more careful of what they offer, since in most instances it would represent their first published work. Second, it would provide the student with copies of his thesis at nominal cost in the form of reprints. This would be very useful for job applications. Even when prospective employers were not sent a reprint by the student they would be able to obtain the thesis series from most libraries, and so could have access to a sample of the student’s work. Furthermore, the faculty would feel more conscientious with respect to the supervision of theses, since it would be evident to other institutions and members of the profession generally what caliber of work was being done. Finally, the work involved could be arranged to accord the students themselves with experiences in publishing in much the same way a law review does in law school. The argument against such a series is that the better theses or redrafts of them will be worth publication in the regular professional journals, and that this would be much preferable. There is also no guarantee that the university series would offer any substantial incentive to high quality, but may well have the opposite effect.*

[22]

The General Form of Graduate Instruction in Economics

These requirements partially dictate the general form of graduate education in economics. For one thing, a certain degree of formality will be required in education at the graduate level. This formality comes about because the entering graduate student usually does not possess the background necessary for graduate work in economics. Unlike the sciences and medicine, it is not practical to require that all entering students possess training in specific areas. The decision by students to become economists almost invariably is made very late in their undergraduate careers, so that it is usually impractical for them to acquire more advanced training in this area while they are undergraduates. Students should, of course, be encouraged to acquire the background at the undergraduate level insofar as possible, and the graduate curriculum may be modified to accelerate students who are adequately prepared. Nevertheless, there will still be a considerable area of the common core to which almost all students should be subjected.

Students who are capable of good work in one direction but find some other area extremely difficult may perhaps be permitted to waive certain of the requirements. The exceptional students, furthermore, need not necessarily be only those brilliant students who excel in economic theory. Students of more specialized interests, such as those primarily interested in the filed of labor, economic history, or corporation finance, should be given consideration fully as much as the theorists.

To a considerable extent, flexibility of graduate training can be secured by more individual attention in the form of some sort of tutorial and/or internship training in graduate school. Such a tutorial and/or internship would make the individual needs of the students known to the faculty, and it would give the student more opportunity to go his individual direction, either filling in gaps in his knowledge or pursuing lines of special interest. It would not always be necessary that senior faculty members be used as tutors. Younger staff members who [23] were themselves more recently graduate students may make more suitable tutors, in that they are closer to recent graduate training and are generally freer with their time.

Finally, it seems necessary to maintain some form of certification as a function of graduate education, as long as the number of students trained is substantial. People hiring students will want to know the kind and caliber of work done by the student in question. It has been suggested that the certification problem can be lessened by relying for purposes of recommendation and scholarship evaluation on more lengthy comments written by the student’s supervisors.

The Period of Graduate Training

It is the present practice of many graduate schools to concentrate the tool courses in the first year of graduate studies. Such an arrangement tends to make a somewhat regimented, formal, and uninspired first year of graduate work. The beginning student is left little room to follow lines in which he is interested or to explore areas to see whether he would find them interesting.

The specialization that takes place in the second year, as noted in the preceding section, often means only a single course in the special field. As a result, a survey course within an area is considered advanced work in that area. This specialization, furthermore, occurs at the same time the student is preparing for his comprehensives, and usually more attention is given to the comprehensives than to the specialization.

The thesis is often not started until after the student has finished his second year of graduate work and passed his comprehensive examinations. As a result, not only the writing of the thesis but the conception of it as well may be done after the student has served his time in residence and left. The consequent lack of supervision, the relegation of the thesis to a part-time task, and the prolongation of the thesis period to a number of years all tend to reduce the quality and usefulness of the thesis.

[24] The panel was generally agreed that the distinction in timing between tool courses, specialization, and the thesis should be less sharp than is current practice. In the first year, the student should be allowed to do some browsing. Some of the tool courses should be postponed until the second year, so that more of a cumulative development in the tools themselves would be possible.

The preliminary work on the thesis should not be put off until the third year of graduate work, and the thesis itself should be completed while the student is in residence. Initial work might start in a thesis seminar in the second year of graduate study. Rather than spending full time on the thesis at any point in his graduate work, the student would be expected to work on his thesis along with other course or seminar work.

Internships, research assistantships, and other such programs may mean that the student will interrupt or prolong the period of graduate work, or he may spend some of his summers in such activities. Programs such as these, however, should be planned in terms of the student’s total graduate training, and should be carried out as part of it. They should not be devised solely in terms of the faculty’s manpower needs—as at present is sometimes the case.

These requirements indicate that a minimum of three years in residence will be required by graduate students to complete the work. Generally speaking, four years will be more usual, so that the student can get practical experience as well as formal training into his graduate training. For the student’s own good, a period of more than five years in residence between entrance and the obtaining of the doctorate is probably undesirable. Should the student contemplate a more ambitious program than this, it should be of a post-doctoral nature. It would be useful for this purpose if universities could set up programs whereby post-doctoral students could obtain internships in business and government for a year, and then return to the university in a teaching position for a year following the internship. Such an arrangement would encourage business and government to take [25] students on an internship basis, and would at the same time give the individual student an opportunity to get established after having served his internship.

Summary and Conclusions

  1. The familiar concept of giving all graduate students in economics basic training in a common core appears to be a useful device, and should be kept as an integral part of graduate training in economics. This common core, if properly conceived, has the advantage of providing some breadth to the student’s training, not only making him more literate, but also giving him a better perspective within which to place his more specialized training. The common core also makes it easier for economists to communicate with each other insofar as they have had the same type of general training. Finally, mobility within the profession is promoted, so that it is possible for economists to move between business, government, and academic work to a much greater extent than might otherwise be so.
  2. The inadequacy of the current training of economists in writing and research was considered to be one of the greatest gaps in graduate training. The ability to express ideas coherently and the ability to carry through research work in a skillful manner should both be considered major tools of the economist. The graduate program, therefore, should take account of both these needs early in the period of graduate training, and attention should continue to be directed to them throughout the graduate program. Both writing and research should be weighted more than is done at present in the grading structure of the graduate program. One of the primary objectives of graduate schools should be to produce people who do not just know, but who can do as well, and the grading structure should be changed to assist in bringing this about. Special programs to promote research training, such as internships in the university or outside of it, should be developed to give the student more research experience under supervised conditions.
  3. Specialization in graduate school should equip the student [26] with more advanced training in various areas. It is important that this training not be too narrowly conceived nor too superficial. Instances where a single advanced course and little outside work is supposed to make a student a specialist are all too frequent. Specialization requires a longer build-up of cumulative work, and may involve going into related areas outside of what is generally considered to be economics. Graduate schools should give more careful attention to the specialized training students receive and whether this training does in fact meet the requirements for genuine specialization.
  4. Graduate training normally takes place over a very extended period. Students often work part time while trying to get their doctorate. It is thought that much would be gained if, as in the case of the professional schools, graduate training in economics could take place in an unbroken period of concentrated effort. If the common core is to be retained as is suggested in item 1 above, and more emphasis is to be placed upon writing, research, and specialization, as suggested in items 2 and 3 above, it seems very probable that the total effort going into graduate training in economics by the student will have to be increased. The concentration of studies into a period of three or four consecutive years on a full-time basis will do much to increase the efficiency of the students’ training and permit these objectives to be met. Summer programs of research or internship training may also be of considerable aid in fulfilling these objectives without extending graduate training further.
  5. The present form of the Ph.D. thesis is not an optimal device for achieving these objectives. It was thought that short theses, which could be reworked more easily and which could generally be made available in published form, would be more manageable and would provide a more effective training device. Such a thesis could be integrated into the graduate training program, and could generally be expected to be written while a student was still in residence; the doctorate would be granted directly upon completion of the period of residence and the thesis.

 

___________________________________

*One panel member has suggested that in cases where a mediocre short thesis is written only an M.A. be granted, and the Ph.D. reserved for theses of exceptional quality.

 

 

Source: John F. Kennedy Presidential Library. Personal Papers of John Kenneth Galbraith, Series 5. Harvard University File, 1949-1990. Box 517, Folder “General Correspondence 8/7/56—12/10/57”.

Categories
Chicago Economists

Chicago. Simons urges the recruitment of Milton Friedman, 1945

 

 

The atomic bomb dropped on Nagasaki was less than two weeks history and the declaration of the surrender of Imperial Japan only five days old. Nothing says “back to business as usual” at the university better than active lobbying on behalf of one’s preferred candidate for an upcoming vacancy, as we see in the following memo for the 33 year old Milton Friedman written by Henry C. Simons to the Chicago economics department chair, Simeon E. Leland. The copy of this memo comes from the President’s Office at the University of Chicago. Simons’ grand strategy was to seamlessly replace the triad Lange-Knight-Mints with his own dream team of Friedman-Stigler-Hart. He feared that outsiders to the department might be tempted to appoint some convex combination of New Dealer Rexford Tugwell and trust-bustin’ George W. Stocking Sr., economists of the institutional persuasion who were swimming on the edges of the mainstream of the time.

Economics in the Rear-view Mirror also has transcribed excerpts from an earlier 77 page (!) memorandum (10 April, 1945) to President Robert M. Hutchins from Simeon E. Leland entitled “Postwar Plans of the Department of Economics–A Wide Variety of Observations and Suggestions All Intended To Be Helpful in Improving the State of the University”.

____________________________

 

Henry C. Simons Urges his Department Chair to Recruit Milton Friedman

August 20, 1945

To: Simeon E. Leland           Economics

From: Henry C. Simons        Economics

 

If Lange is leaving, we should go after Milton Friedman immediately.

It is a hard choice between Friedman and Stigler. We should tell the administration that we want them both (they would work together excellently, each improving what the other did), Friedman to replace Lange, Stigler to replace Knight and to be with us well ahead of Knight’s retirement. We might also say that we want Hart to replace Mints at Mints’s retirement, and also to be with us in advance, but are happy to have him financed by C.E.D. [Committee for Economic Development] for the present.

Yntema evidently is thinking of getting Friedman shortly. We should exploit this possibility. Milton has now a great yen for a University post and would probably turn down an offer from C.E.D., even at much financial sacrifice, if a good academic post were the alternative (as it might be, at Minnesota or elsewhere). He is rather footloose—not anxious to go back either to the Treasury or to the National Bureau. We should grab him now, offering temporary joint appointment with C.E.D. and full-time, permanent appointment when he is through with C.E.D.

Friedman is young, flexible, and available potentially for a wide variety of assignments. He is a first-rate economic theorist, economic statistician, and mathematical economist, and is intensely interested over the whole range of economic policy. He has been outstanding in every organization where he has worked—here with Henry Schultz, at the National Bureau, at the Treasury, and now recently in the Army project at Columbia. Moreover, he is one of those rare cases of able young men who have enjoyed large experience and responsibility in Washington without being at all disqualified thereby for academic work.

The obvious long-term arrangement is a joint appointment with the Cowles Commission. Marschak would, I’m sure, like to have him; and Milton would like to settle into a major project of empirical research, e.g., on enterprise size and productional efficiency. Bartky may be expected strongly to support the appointment, for its strengthening of the University in statistics. The School of Business could well use Milton, to give its few advanced courses in statistics, if Yntema continues to price himself out of the University. Moreover, Milton probably would be delighted to work partly in the Law School, and be extremely useful there. In the Department, he would be available for statistics, mathematical economics, pure economic theory, taxation, and almost any field where we might need additional courses.

If University officers want outside testimony, they could get it from Randolph Paul or Roy Blough (as regards the Treasury), from Arthur F. Burns (National Bureau), from Abraham Wald, Allen Wallis, and Barky (as regards war research), and from Bunn at Wisconsin (as regards possible usefulness to the Law School)—not to mention George Stigler, Harold Groves, Wesley Mitchell, Simon Kuznets, Erwin Griswold, et al.

Perhaps the best thing about Milton, apart from his technical abilities, is his capacity for working as part of a team. He is the gregarious kind of intellectual, anxious to try out all his ideas on his colleagues and to have them reciprocate. He would doubtless be worth his whole salary, if he neither taught nor published, simply for his contribution to other people’s work and to the Department group as a whole. But he is also intensely interested in teaching, and far too industrious not to publish extensively. Our problem would be not that of finding ways to use him but that of keeping him from trying too many tasks and, especially, of leaving him enough time for his own research.

It would, I think, be good policy and good tactics to submit a major program of appointments, including [Frank W.] Fetter, Friedman, Stigler, Hart, and an economic historian (Innis or Hamilton), in the hope of getting them all within a few years, some on joint appointments with, notably, the Cowles Commission, the Law School, the School of Business (?) and, temporarily, the C.E.D. Research Staff. Such a program would serve to protect us against administration pressure for less good appointments (e.g.,  Stocking [George Ward Stocking, Sr., Ph.D. Columbia, 1925]), and from Hutchins’s alleged complaint that, while he wanted to consider major appointments in economics, the Department simply would not make recommendations. We should, in any case, err on the side of asking for more appointments than we can immediately get. Otherwise, available funds may go largely elsewhere—e.g., into Tugwell-like, lame-duck appointments, and into Industrial Relations, Agricultural Economics, and other ancillary enterprises, at the expense of the central field of economics.

There is, I trust, substantial agreement within the Department, on the men mentioned above. This fact, if fact it is, should be made unmistakably clear to the administration.

Incidentally, if we are going to explore possibilities of an appointment in American economic history (and I’m probably alone in opposing), we should do so only in co-operation with the History Department and with (from the outset) joint plans for joint appointments.

 

HCS-w

 

Source: University of Chicago Archives. Office of the President. Hutchins Administration. Records. Box 73, Folder “Economics Dept., 1943-45”.

Image Source: University of Chicago Photographic Archive, apf1-07613, Special Collections Research Center, University of Chicago Library.

Categories
Chicago Exam Questions

Chicago. Exams for Introduction to Money and Banking, A. G. Hart, 1932-35

 

 

In an earlier post I provided the course outline and readings for the first money and banking courses taught by Albert Gailord Hart during the depths of the Great Depression. Today’s post provides transcriptions of the final examination questions for the course. Interesting to note that the course final exam was spread over two days in 1934 and 1935.

 

_______________________________

Course description

[Economics] 230. Introduction to Money and Banking.—The material in the course includes a study of the factors which determine the value of money in the short and in the long run; the problem of index numbers of price levels; and the operation of the commercial banking system and its relation to the price level and general business activity. Prerequisite: Social Science I and II or equivalent.

Source: University of Chicago, Announcements [for 1933-34], Arts, Literature and Science, vol. 33, no. 8 (March 25, 1933), p. 266.

_______________________________

Econ 230
A. G. Hart

FINAL EXAMINATION, DECEMBER 21, 1932

Answer questions I, II and III.

I. (About 20 minutes).

Suppose a large manufacturing firm wants more capital. It might establish a bank with $1,000,000 capital paid in in cash and $1,000,000 in deposits transferred from other banks. Apart from legal restrictions on the amount a bank may lend to a single borrower, could the manufacturing firm borrow $20,000,000 from the bank (reckoning 10% reserve)? If not, how much could be obtained from such a bank? Explain.

II. Answer all four parts, allowing about ten minutes for each:

a) Explain the difference between a sight draft and a cable draft in foreign exchange. Which includes an interest charge? Why?

b) Suppose demand depositors of the First National Bank of Chicago transferred $1,000,000 from demand to time deposits. What would be the change in the amount of reserve deposits which the First National is required to hold at the Federal Reserve? What would be the change in required reserve brought about by a similar shift of deposits in a state bank, member of the Federal Reserve System, in Cleveland, Ohio?

c) Explain what is meant by open market operations. How do they affect the money market?

d) Define Mr. Hawtrey’s concepts of “consumers’ outlay” and “unspent margin”. How do they figure in Mr. Hawtrey’s theory of the price level?

III. Answer any two parts, allowing about twenty minutes for each:

a) Explain the difference between the price level defined by the Fisher form of the quantity equation and a cost-of-living index for the working class. What might cause these two price levels to behave differently?

b) If counterfeiters succeeded in making perfect reproductions of Federal Reserve Notes and placed $100,000,000 in circulation, how would this differ from 1) an expansion of $100,000,000 in bank loans, 2) an extra $100,000,000 in greenbacks used by the government to pay unemployment relief in the following respects: i) effect on prices; ii) effect on the total volume of production and employment; iii) effect on the direction of production; iv) “forced saving”? Give reasons.

c) If citizens of a country increase their investments abroad, what influence will this have 1) on the price of sight bills on a foreign country; 2) on the balance of trade; 3) on the prices of domestic goods in the first country? Why?

d) What is the basis of distinction between “real” and “monetary” theories of the business cycle? Mention and criticise an example of each type.

_______________________________

Econ 230
A.G. Hart

Hour Examination, August 3, 1933

 

Answer questions I, II, and III

  1. Bank Statement:

The following items make up the condensed statement of one of the great New York banks for two recent call dates (to nearest $1000):

Item June 30, 1931 June 30, 1933
(000 omitted)
1. Stock of Federal Reserve Bank
2. Undivided Profits
3. U. S. Government securities
4. Other bonds and securities
5. Dividend payable July 1
6. Customers’ acceptance liability
7. Capital
8. Acceptances
9. Real estate
10. Reserve for contingencies
11. Deposits
12. Cash and due from banks
13. Surplus
14. Other assets
15. Other liabilities
16. Loans and discounts
17. Total resources
18. Total liabilities
$8,880
25,581
281,786
174,500
7,400
169,255
148,000
174,252
35,036
14,720
1,897,544
531,352
148,000
3,030
80,828
1,295,486
2,499,325
2,499,325
$8,160
8,705
207,955
246,845
2,590
91,443
148,000
93,354
32,069
3,334
1,408,337
351,374
50,000
15,466
18,747
779,755
1,733,067
1,733,067

A. Reconstruct the statement, separating assets from liabilities.
B. Which of the above items represent the investment of stockholders in the back? Do you think the total of these items bears a normal relation to total resources?
C. Does any of the above items show the bank’s primary reserves? If not, try to estimate their amount. Compare primary reserves with deposits. Do you think the proportion shows the bank to be healthy? Explain.
D. Which of the asset items consist wholly or in part of “secondary reserves”?
E. What items would replace #12 in a more detailed statement?
F. Suggest explanations for the decrease between 1931 and 1933 in items 11, 8, 5, 16, and 13.

 

  1. Federal Reserve:

A. What is the “open market committee”?
B. List three of the more important powers of the Federal Reserve Board over the Federal Reserve Banks.
C. Name five cities having Federal Reserve Banks

 

  1. Quantity Theory

It is the announced policy of the Roosevelt administration to spend about $3,000,000,000 within the next year on public works, raising the funds by borrowing from the Federal Reserve and member banks. In what sense is this “inflation”? Assuming no inflationary or deflationary actio from other sources, how much might this program be expected to raise the “general price level” in the long run? Explain.

 

_______________________________

FINAL EXAMINATION
Economics 230
Summer Quarter 1933

(follow link above)

_______________________________

 

FINAL EXAMINATION
Economics 230
Winter Quarter 1933

I
(About 30 minutes)

The following was the consolidated statement of the twelve Federal Reserve Banks for March 1, 1933 in abbreviated form:

Item March 1 Feb. 21, 1933
(000 omitted)
1. Total gold reserves
2. Total Reserves
3. Discounts secured by U.S. obligations
4. Other discounts
5. Total bills discounted
6. U. S. securities
7. Total bills bought
8. Federal Reserve notes in circulation
9. Total deposits
10. Reserve ratio against notes and deposits
$2,892,083
3,066,537
418,921
293,470
712,391
1,835,963
383,666
3,579,522
2,157,190
53.5%
$3,118,393
3,304,644
105,102
222,036
327,138
1,834,233
179,576
3,000,248
2,399,398
61.2%

Answer parts a) to d): a) Which of the above are asset items in which liabilities? What items are missing which would appear the complete statement? b) What makes up the difference between items 1 and 2 from March 1? c) Explain the changes in items 1, 5, 6, 7, 8 and 9 in terms of the conditions of the week covered, paying special attention to interrelations of the changes. d) Calculate free gold under the regular rules and under the Glass-Steagall Act (assuming notes issued not in circulation to be $100,000,000), as of March 1.

 

II

Answer all three parts, allowing about ten minutes for each:

a) Explain what is meant by open-market operations by the Federal Reserve Banks. Under whose authority are they conducted? What is their effect on the money market?

b) Explain the method of calculating “net demand deposits” for working out the required reserves of member banks.

c) Write out the Fisher equation of exchange and define the meaning of the symbols used. (Criticism or discussion not called for.)

 

III

Answer any two parts, allowing about twenty minutes for each:

a) Distinguish between “real” and “Monetary” theories of the business cycle. Mention and criticise an example of each.

b) Discuss: “The very process of financing increased production puts into circulation enough money to buy the added output, so that supply and demand must be equal. After all… trade is but a perfected system of barter.”

c) “In these days of serious world-wide maladjustments the importance of economic stability is likely to be over – rather than underrated.” Discuss.

d) Indicate the advantages and shortcomings of the quantity theory of money 1) for short-run analysis, 2) for predicting long-period tendencies.

_______________________________

 

ECONOMICS 230
Final Examination, Mch. 22-23 [1934]

Part I – answer questions 1 – 3 and either 4 or 5

  1. If the Federal Reserve wishes to diminish the reserves of the member banks, what can it do? Can anything happen to make these measures ineffective? If so, what?
  2. What is a letter of credit?
  3. What differences in meaning are there between the price level of Keynes’s first equation and that of Fisher’s equation?
  4. M. (100%) Nichols, of the First National Bank of Englewood, recently wrote to the R.F.C.: “when I believe that our merchants can safely and profitably borrow money, with a reasonable assurance of paying it back, I shall tell them so… I refuse to take this responsibility as I do not believe this is a safe time either to borrow or to loan.” Discuss this in relation to the government’s claim that refusal to expand bank loans is retarded recovery.
  5. It has been said that the effects of inflation are primarily on the distribution of wealth, those of deflation on its production. Discuss.

 

Part II – Answer questions 6-8 and either 9 or 10.

  1. Distinguish between F. R. Notes and F. R. Bank notes.
  2. Explain the meaning of “velocity of circulation”.
  3. Would the following tend to raise or lower the prices of foreign-currency units in dollars: a) increased demand for sugar in this country? b) an increase in our tariff duties on English textiles? c) resumption of payments to our government on account of war debts? d) the rise of wage rates in this country brought about by NRA? Explain briefly in each case.
  4. Do you think that the Roosevelt monetary policy will succeed in raising prices appreciably? Why and How? If you do, what do you think will be its effect on the following price relationships. Salaries vs. cost-of-living? Wages vs. cost-of-living? Farm prices for crops vs. prices of things farmers buy? Explain.
  5. Which of the following groups have most to gain by inflation and which least: policeman? Owners of mortgaged down-town real estate? Exporters? Railway bondholders? Railway stockholders? Wage earners? Unemployed steelworkers? Explain in each case, and if you cannot tell whether the group would gain, explain why you cannot.

_______________________________

Econ. 230
A. G. Hart

Final Examination
December 19-20, 1935

  1. Gold imports into the United States in the 22 months ending October 31, 1935 totaled nearly $2473 million (new valuation), increasing our monetary gold stocks by about one third. a) Suggest explanations for the movement. b) Estimate the effects of the inflow of total reserves of member banks; on their excess reserves. Explain your reasoning. c) Estimate the effects of the inflow on total reserves and on excess reserves of the Federal Reserve Banks, and explain.
  2. If American monetary policy brings about a substantial rise of prices within the next five years, how will this affect the interests of a) a widow with an annuity from a life insurance company; b) a railway engineer; c) a university professor; d) an unemployed carpenter; e) a postal clerk; f) an automobile mechanic. Give grounds for your answers.

 

 

  1. State and criticise the views of Gregory on the merits of the American devaluation from an international standpoint.
  2. Describes a means by which the American monetary authorities could act to stabilise: a) the dollar price of a foreign gold-standard currency, b) the volume of checking deposits in the hands of the public, c) an index number of wholesale prices. In each case what reasons are there for doubting the effectiveness of these means?
  3. (Optional – write only if time permits.) As among the three sorts of “stabilisation” mentioned, which would you prefer to see made the guide of monetary policy, and why?

 

Source: Columbia University Archives. Albert Gaylord Hart Papers. Box 61, Folder “Assignments and Other Memoranda for Reserve in Harper Reading Room Econ 230, A. G. Hart”.

Image source: Ibid.

 

 

Categories
Courses Exam Questions Toronto

Toronto. Honors Exam. Money, Credit and Prices. 1933

_______________________________

The honors examination questions for Money, Credit and Prices from the University of Toronto transcribed below were filed away by A. G. Hart in a folder marked “Chi[cago] Qualifying”, perhaps not an ideal resting place for this particular archival artifact. At least now these exam questions are discoverable through a standard internet search and provide researchers going to the University of Toronto archives a tip should they search for economics course materials there.

_______________________________

Course descriptions

3e. Money, Credit and Prices. A course dealing with monetary theory and related subjects, including the discussion of the role of money in economic theory; bimetallism; the gold standard; the gold exchange standard; the relation between money, credit, production and prices; the business cycle; central banks and the control of credit; stabilization of business; the foreign exchanges; the role of money in the theory of international trade; money and foreign exchange; problems in various countries, including reparations. For reference: Cassel, Theory of Social Economy, Vol. II, and Money and Foreign Exchange after 1914; Fisher, The Purchasing Power of Money; Keynes, A Treatise on Money; Marshall, Money, Credit and Commerce; Edie, Money, Bank Credit and Prices; Willis and Beckhart, Foreign Banking Systems; Burgess, Interpretations of the Federal Reserve Bank; Mitchell, Business Cycles, the Problem, and its Setting; Snyder, Business Cycles and Business Measurements; Hobson, Rationalization and Unemployment; Gregory, Foreign Exchange; Taussig, International Trade; Angell, International Prices; The Young Plan; Reports of Agent General for Reparations; Reports of League of Nations Gold Delegation; The Macmillan Report, 1931; Current Financial Literature. Three hours a week.

3h. Banking. A special course on the theory and practice of banking operations. One hour a week.

 

Source: University of Toronto Calendar, Faculty of Arts 1932-33. University of Toronto Press, pp. 112-113.

Image Source: Detail from photo of A. F. Wynne Plumptre (1972) from the University of Toronto Archives Image Bank.

_______________________________

Card paper clipped to examination copy

I thought you might find this of interest.
A. F. Wynne Plumptre [B.A., Lecturer]
Kings College, Cambridge
Toronto, Canada

_______________________________

 

UNIVERSITY OF TORONTO
FACULTY OF ARTS

ANNUAL EXAMINATIONS, 1933

THIRD YEAR—HONOUR

ECONOMICS 3e, 3h
MONEY, CREDIT AND PRICES

Examiners—The Staff in Economics

 

(Question ONE must be answered by all candidates, and THREE or FOUR other questions.)

  1. What do you mean by “inflation”? Under what circumstances, if any, is it desirable?
  2. The following figures appear in the monthly returns of the combined Canadian Chartered Banks:

(in millions of dollars)

Sept. 1929. Sept. 1932.
Current Commercial loans

1,404

1,003

Total Securities held

487

704

Demand Deposits

759

481

Notice Deposits

1,471

1,359

Bank Notes in circulation

197

132

Finance Act borrowings

79

23

Sketch the probable causes of these movements.

  1. It is said, often in criticism of French financial methods, that the power of finance is used in that country to further political ends. In England, on the other hand, efforts have usually been made to keep “politics” dissociated from “finance”; i.e., to keep politicians from dictating the country’s monetary and financial policies. How far, in your opinion, can or should the two be kept separate in Canada or any other country?
  2. “The establishment of the federal reserve bank system…is actually the reason why they have had the recent trouble in the United States banks.” (Sir John Aird, quoted in the Toronto Daily Star, March 22nd, 1933.) How far do you agree with this statement?
  3. In maintaining the gold standard, “world wide international co-operation becomes all but essential just at the moment when the particular local manifestations of the universal trouble occupy the whole attention of the Government in each country and make international action specially difficult.” (Sir Basil Blackett.) Is this a fair summary of the causes of the breakdown of the international gold standard? If so, does it necessarily follow that the restoration and subsequent maintenance of the gold standard is impracticable?
  4. Give an outline of what is meant by any two of the following policies:

Bimetallism,
Remonetization of silver,
Revaluation of gold,
Reduction of central bank reserve ratios.

  1. Outline very briefly the theory of “comparative costs” in international trade. How far do you think it is desirable that members of the newly appointed Canadian tariff board should be familiar with the principles of this theory?
  2. Do you believe that monetary policy is, or might be, a major factor in determining the level of prices and prosperity in either Canada or England or some other country? (Candidates should answer this question with respect to one country only.)
  3. “Booms and slumps are simply the expression of the results of an oscillation of the rate of interest about its equilibrium position.” (J. M. Keynes.) How far do you agree?
  4. Suppose that, at the forthcoming World Economic Conference, it were generally agreed that international exchange rates should be stabilized immediately. What factors would you then take into consideration in estimating at what rate the Canadian dollar should be stabilized? How far would the theory of “purchasing power parity” assist you?
  5. It appears to be customary for monetary theorists to make use of equations in explaining their theories. Why do you think they have used this method? Do you think that such an equation is likely to clarify or becloud the theory to which it refers?

 

Source: Columbia University Archives. Albert Gailord Hart Papers, Box 60, Folder “Exams: Chi[ago] Qualifying”

 

Categories
Courses Economists Exam Questions Harvard

Harvard. Economics of Corporations. Dewing and Opie, 1929

 

Professor Arthur Stone Dewing (1880-1971) and Dr. Redvers Opie taught the course Economics of Corporations at Harvard that was given in the second semester of the academic year 1928/29.  In an earlier posting I transcribed a student review of the course that was published in the Harvard Crimson (December 11, 1929).

Dewing was born (April 16, 1880) and died (January 19, 1971) in Boston. His academic degrees were awarded by Harvard (A.B., 1902; A.M., 1903; Ph.D. in Philosophy, 1905). Dissertation title: “Negation and Intuition in the Philosophy of Schelling.” He also studied at the University of Munich. Dewing taught philosophy from 1902 to 1913 and in economics and finance from 1911 to 1933.  He was one of the founders of the Harvard Business School. For a memorial see Cornelius Vermeil, “Arthur Stone Dewing”, Proceedings of the Massachusetts Historical Society, Third Series, Vol. 83, 1971, pp. 165-167.

Incidentally Arthur S. Dewing was a distinguished numismatist, his collection of nearly three thousand ancient Greek coins was considered “one of the most outstanding in the hands of a private collector in the world.” Here a fascinating article about the theft and recovery of much the Dewing collection.

Redvers Opie (1900-1984) was a Harvard economics Ph.D., best known as the translator of Joseph Schumpeter’s The Theory of Economic Development (1934). From the “Company Info” page of Ecanal (Economic Analysis for Company Planning in Mexico) with information added by me regarding the dates of Opie’s academic degrees:

Ecanal was founded in 1976 by the British economist, Dr Redvers Opie, who was educated at Durham University [B. Com., 1919]. He taught at Oxford University and became the Bursar of Magdalen College. Later on he received a PhD from and taught at Harvard University [A.M. in 1927. Ph.D. in 1928. Thesis: “John Stuart Mill: a Reexamination”]. On the recommendation of John Maynard Keynes, he became the UK Treasury representative in Washington DC, and later on one of the five members of the UK delegation to the Bretton Woods Conference, which gave birth to the IMF and the World Bank. He started Ecanal upon becoming a naturalized Mexican as the source of critical analysis of the economy and government policy useful for business.

Cf. the 1933-34 Additional Readings for General Examination Corporations for Harvard.

Note: In the Course Announcements for 1928-29 (second edition), p. 122, Dr. C. E. Persons was originally scheduled to teach this course.

The information for this course comes from the course notes taken by later University of Chicago and Columbia University economist, Albert Gailord Hart. Hart’s handwriting defies encryption though I am proud to say that all but two or three words in what follows has been successfully deciphered. The course reading assignments are followed by the final examination questions.

__________________________________

Economics 4b
Assignments

Buy Jones—Trust Problem

[Jones, Eliot. The Trust Problem in the Unites States. New York: Macmillan, 1929. For the 1921 edition]

Choose one[:]

Pollock & Maitland—Law – Vol. I 586-518

[Pollock, Sir Frederick and Frederic William Maitland. The History of English Law before the Time of Edward I, 2nd ed. Cambridge (England): Cambridge University Press, 1923.]

[Illegible name, “Bold—-“?] 1st ed 362-376 [;] 3rd ed 469-490

J. P. Davis Corp. 1, 7 (2 & 8 optional)

[Davis, John P. Corporations; A Study of the Origin and Development of Great Business Combinations and of their Relation to the Authority of the State. New York: G. P. Putnam’s Sons, 1905. Volume I, probably Chapter I (Introduction, pp. 1-12) and Chapter II (The Nature of Corporations, pp.13-34) intended; Volume II, probably Chapter VII (Legal View of Corporations, pp. 209-247) and Chapter VIII (Modern Corporations, pp. 248-280) intended]

Baldwin Mod Pol Inst

[Baldwin, Simeon Eben. Modern Political Institutions. Boston: Little, Brown, and Company, 1898]

Buy one[:]

Dewing A. S. Cor. Finance (simplified) omit 8, 17-19, 21

[Dewing, Arthur Stone. Corporation Finance. New York: Ronald Press, 1922]

Lyon, Hastings—[Cor. Finance] (specialized) omit I 6,8, II 4

[Lyon, Hastings, Corporation Finance. Boston: Houghton Mifflin, 1916
Part I: Capitalization. Part II: Distributing Securities, Reorganization.]

Hour exam in mid-March, another early April

Jones 1-5, 9, 11-2, 14-6

Watkins Indust Combin & Pub. Pol. 11 (R 223-47)
Not held for cases except big ones.

[Watkins, Myron Webster. Industrial Combinations and Public Policy: A Study of Combination, Competition and the Common Welfare. Boston: Houghton Mifflin Company, 1927.

Jones 17-18

Take one.

1 Indust Conf Bd “Trade Associations”

[National Industrial Conference Board, Trade Associations; Their Economic Significance and Legal Status, 1925.  Reviewed in Journal of Political Economy, Vol. 35, No. 3, June, 1927. pp. 428-30.]

2 [Indust. Conf. Bd.] “Public Reg. of Competitive Practices”

[(Myron W. Watkins), National Industrial Conference Board, Public Regulation of Competitive Practices, 1925. Revised and enlarged edition, 1929. Third edition, 1940.]

3 [Illegible word] the Law plus Watkins

4 Kirsch—“Trade Ass’ns.

[Kirsch, Benjamin S. Trade Associations: The Legal Aspects. New York: Central Book Co., 1928. Review in Journal of Political Economy, Vol. 38, No. 2 (April, 1930), pp. 238-240.]

Ad lit. A. Smith V, ch. I, Pt III, article I, 211-245.

Geneva Economic Conference of 1927. Publications on Cartels.

[Paul de Rousiers, Cartels, Trusts, and Their Development; D. H. MacGregor, International Cartels. Geneva, 1927]

 

Source: Albert Gailord Hart Papers. Box 60, Folder “R Opie 1929 Monopoly etc”.

__________________________________

 

1928-29
HARVARD UNIVERSITY
ECONOMICS 4b

PART I

(About one hour)

Write an essay on one of the following topics:

(a) Public Policy and Business Standards.
(b) Trade Association Activities and the Competitive System.
(c) The Rule of Reason.
(d) Government Control of Combination and Consolidation To-day.

 

PART II

Answer not more than FOUR questions.

  1. What significance has a study of “laws of return” for an understanding of the problems connected with industrial combinations?
  2. “Certainly the implication of the Webb Act is that enforced competition is too weak, too inefficient to meet monopolistic combination. The question may be fairly asked, what does this admission entail in regard to our domestic trust policy?” Defend, refute or modify.
  3. Does the establishment of the Federal Trade Commission reflect any changes in the relation of Government to industry? What have been the most important activities of the Commission since its inception?
  4. What are the chief causes and purposes of corporate reorganization? Describe the usual procedure adopted, paying particular attention to the methods of protecting the interests of the various parties involved.
  5. “The trust dissolutions have not resulted in a spectacular and instant rescue of the consumer from the evils of monopoly, but that was hardly to have been expected.”
    Do you agree?
    What has been accomplished by trust dissolutions?
  6. “The stockholder has a right to receive the earnings of the corporation as dividends; and the existence of a large surplus simply shows that the stockholder has been deprived of his rightful income.” Does this indicate an intelligent understanding of corporate surplus?

Final. 1929.

 

Source: Albert Gailord Hart Papers. Box 60, Folder “Exams: CHI Qualifyin[?]”.

Image Source: (Dewing, left) Harvard Album 1925; (Opie, right) Harvard Album 1932.

Categories
Chicago Courses Suggested Reading Syllabus

Chicago. Introduction to Money and Banking. A. G. Hart, 1933

In the early 1930’s Lloyd Mints alternated teaching the undergraduate money and banking course at the University of Chicago (Econ 230) with the doctoral student Albert G. Hart who held the ranks of teaching assistant/instructor before receiving his Ph.D. in 1936. In Hart’s papers at Columbia University there is a copy of material for his course kept for students in the Harper Reading Room at the University of Chicago. The “Report on Conduct and Content of Course” included in this posting presents a detailed outline of his course as of its third iteration. Square brackets are used where I have supplied specific page numbers for the textbook assignments that I have found elsewhere in the same folder with this Report. 

In the Review section Hart includes among the “leading ideas” of the course: “… in fields where prices are sticky, inflation and deflation take themselves out on volume of sales and hence of production and/or employment…”

______________________

If you find this posting interesting, here is the complete list of “artifacts” from the history of economics I have assembled thus far. You can subscribe to Economics in the Rear-View Mirror below. There is also an opportunity for comment following each posting….

_________________________________

[Course Description from University of Chicago Announcements]

230. Introduction to Money and Banking.—The material in the course includes a study of the factors which determine the value of money in the short and in the long run; the problem of index numbers of price levels; and the operation of the commercial banking system and its relation to the price level and general business activity. Prerequisite: Social Science I and II or equivalent. Summer, Hart; Autumn, Mints; [Spring], Hart.

Source: University of Chicago, Announcements [for 1933-34], Arts, Literature and Science, vol. 33, no. 8 (March 25, 1933), p. 266.

_________________________________

 

Report on Conduct and Content of Course.

Econ 230
A. G. Hart

General Remarks:

My general policy (in which I follow the suggestions of Mr. Mints) has been to conduct the course definitely as a theoretical study, with the emphasis on analysis rather than history or factual description. Class meetings are devoted largely to discussion, with inevitable lapses into lecturing when historical or factual questions cannot be avoided. My reading assignments have been light (about 100 pages a week), but are of a character to call for mulling over. So far as possible I have avoided second-hand textbook material and gone to authoritative writers. On the necessary technical banking material a textbook is inevitable, and I have used Rodkey’s Banking Process twice and Bradford’s Banking once. (To my taste the former is definitely preferable.) On other subjects I have gone to standard authorities, as Irving Fisher, J. M. Keynes, Alvin Hansen, Wesley Mitchell, D. H. Robertson. These writers are emphatically not beyond the grasp of undergraduate classes here. Some points have been emphasized by written exercises; and I have tried by giving our examinations and by putting past examinations on reserve to make my examinations a guide to what I expected students to take away with them. Full assignments, as made this summer, and past examinations are in my assignment folder on reserve in Harper main reading room.

The following scheme is that given in my assignments; the order of various points somewhat altered from time to time, especially when current events bring certain sides of the course to the fore.

 

Scheme of Course:

I.  Introduction: Money Concepts; Sketch of Monetary History; Present Circulating Medium in U. S.

Explanation of purposes of the course; the notion of money in the broader notion of circulating medium; a summary of the history of the American circulating medium; brief analysis of the current statistics on money in circulation and on bank deposits. Emphasis is laid on the fact that the class of circulating media has no clear-cut boundary, but shades off into the realm of ordinary commodities, and on the importance of bank deposits in American circumstances. Reading: Robertson, Money, pp. 1-17. Class Time: usually two, perhaps three hours.

II. The Quantity Theory of Money According to Fisher.

An analysis of the relations between quantity of money and bank deposits, their velocities, prices, and volume of transactions, through the Fisher equation. Emphasis is laid on the validity of the equation, the restricted conditions under which the Quantity Theory holds, and the consequent inadequacy of the Quantity Theory for short-period analysis; also on relative flexibility of different groups of prices. Reading: Fisher, Purchasing Power of Money, pp. 1-32, 47-54, 74-111, 184-97. Class Time: five or six hours.

III. The Quantity Theory According to the Cambridge School; European Postwar Monetary Experience.

A restatement of the Quantity Theory in “Cambridge” terms, using Keynes’ Monetary Reform as type. This is used to reinforce the analysis of the difficult “velocity” problem in the preceding section, and leads into some examination of the workings of inflation. The idea of forced saving is here brought out. Lately I am coming to doubt the pedagogical usefulness of the formulation in Monetary Reform; and I may take a different tack another time. Reading: Keynes, Monetary Reform, pp. 3-45, 46-80 [, 81-94]; Robertson, Money, e.g. 18-43. Class Time: three or four hours.

IV. Index Numbers: Refinement of Concept of Purchasing Power.

The concept of an index number of prices as reflecting changes in the value of a composite commodity of fixed make-up; index of quantity as reflecting valuations of different mixtures of goods at fixed prices. Necessity of index number analysis to put meaning into notion of price and output changes brought about by money. Emphasis is placed, of course, on fundamental notions and interpretation, not on the technique of index-number compilation. Reading: Fisher, Purchasing Power, pp. 198-233; memorandum of my own in assignment folder. I should assign Keynes [Treatise (optional), I, pp. 53-94] but for library difficulties. Written Assignment: problem in index-number computation, with simplified arithmetic, calculated to bring out differences among the various basic formulas. Class Time: three or four hours.

V. Nature of Banking and Clearing.

(In the Autumn Quarter, 1932 I put this before the treatment of monetary theory, which appears to be just about as satisfactory as the present arrangement.) Basic character of a bank; demand liabilities greater than quick assets, and demand liabilities serving as circulating medium. Mechanism of making and transferring deposits; clearing relations among banks. History of American banking. Study of bank reserves; of earning assets. Principle of credit pyramiding on increment of reserves. Reading: textbook passages [Rodkey, Banking Process, 1-86, 124-37, 137-54, 178-95, 201-224, 267-86; or Bradford, Banking, pp. 1-51, 52-96, 98-123]; Phillips, Bank Credit, chapter III [pp. 32-77]. Written Assignment or Hour Examination: reconstruction and interpretation of dismembered bank statement, involving comparison of two banks or one bank at two dates. Class Time: ten or twelve hours.

VI. Central Banking: Federal Reserve System.

Analysis of open market, re-discount and clearing functions of the Federal Reserve Banks, and of the administration of the system and its influence on the member bank reserve position. I have made no attempt to cover foreign central banking except for an hour’s lecture on the Bank of England and incidental references from this point in the course on to practices of foreign central banks. This is not because I doubt the usefulness of treating foreign banking ceteris paribus; but the marginal utility of time in a one-quarter course is so high as to put this below the zone of profitable use of resources. Emphasis is placed on the essential simplicity of central banking — holding cash reserves and earning assets and swapping them on the open market according to the position of business—, on its necessary non-profit character, etc. The leading ideas in this section are largely modeled on Mr. Mints. One of the drawbacks of using Bradford as text is the necessity of scrambling sections V and VI together to follow its arrangement. Reading: textbook passages [Rodkey, Banking Process, pp. 36-123, 196-200, 224-238; or Bradford, Banking, pp. 125-144, 145-168, 170-222, 223-247, 248-274, 311-336, 337-355]; W. R. Burgess, Reserve Banks and the Money Market, pp. 206-229; passages in Federal Reserve Bulletin [March 1932, pp. 1-5]. Written Assignment or Classroom Work: Analysis of weekly statements of Federal Reserve Banks and weekly reporting member banks; calculation of Federal Reserve reserve and collateral requirements and of free gold. [take F.R. statement for July 13 (published in morning papers July 21). Compute gold reserve requirements against deposits and notes, calculate excess gold reserves. Compute gold collateral requirement on notes, with and without Glass-Steagal arrangement, and calculate free gold on each basis.] From this point in the course the statement is discussed at some length every Friday. Class Time: five or six hours, plus a short time each week toward the end of the quarter for study of the statement.

VII. Foreign Exchange; International Banking Relations.

The mechanism of the exchange markets: acceptances, cable drafts, etc. The balance of payments and supply and demand of foreign funds; determination of the rate and adjustment of balances on and off the gold standard. Classical mechanism of adjustment; orthodox types of central bank intervention; post-war manipulative tricks in exchange market. Due largely to the trend of current events, emphasis has been placed on the determination of rates off the gold standard and on the short-run importance of flight from the currency and other types of speculative movements. Reading: as the topic is peripheral and cannot absorb much time, I have compromised between textbook readings and authoritative material, and have been assigning Taussig, Principles, vol. I, pp. 447-78 besides text-book passages. [Rodkey, Banking Process, pp. 155-68; or Bradford, Banking, 275-310; Robertson, Money, 69-91] Class Time: three or four hours.

VIII. Short Period Monetary Theory.

Inapplicability of quantity theory (though not of Fisher equation) in “transition period”. Generalization of Fisher cash-transactions approach: can be used wherever flow of goods and flow of money which buys it can both be conceptually isolated. Analysis through income concepts. Hawtrey’s scheme as a special case of transactions approach; illustration of analysis by his cycle theories. Concept of forced savings. Reading: Fisher, Purchasing Power, 55-74; Robertson, Money, 92-116; Hawtrey, Currency and Credit, 1-64. Class Time: four to six hours.

IX. Business Cycles.

Concept of business cycle; basic types of series showing cycle. Types of business cycle theories. Problems of “overproduction” in various senses. Reading: Mitchell, Business Cycles, the Problem and its Setting, pp. 1-60. Class Time: three to five hours.

X. Banking Policy and “Stabilization”.

General concept of stabilization – largely connotation with meaning vague. Monetary stabilization – constancy of index number or the exchange rate. Partial incompatibility of former and latter. Criteria of desirable indices to stabilize – full employment chief – point to desirability of gently raising money wages on average; consequences in other price groups. Central difficulty of price stickiness. Ways and means – inadequacy of central-bank control; inevitable influence of government finance. Savings-investment criterion of policy – its weaknesses. Reading: Robertson, Money, 144-194; Hansen, Economic Stabilization in an Unbalanced World, 3-27, 65-113, 271-314. Class Time: 4 to 6 hours.

Review:

When possible (it isn’t in the summer) I plan to devote four or five hours to systematic review. Leading ideas are brought out more clearly; especially: 1) the price level associated with the given flow of goods cannot rise, unless less goods of this group are sold or more money is spent on them; 2) when price movements are on foot the various forms of “price stickiness” make the movements of different speed and amplitude in different fields; 3) it is these relative price movements which are of practical importance; 4) in fields where prices are sticky, inflation and deflation take themselves out on volume of sales and hence of production and/or employment; 5) a banking system uncontrolled except by “qualitative” considerations will inevitably bring about inflation and deflation involving destructive movements of relative prices and of production; 6) in fact these considerations do not even enable a banking system to protect itself against a heavy proportion of failures in deflation.

 

Discussion of Above Scheme:

It will be observed that the course as I have given it contains little or no discussion of two topics which occupy a great deal of the literature, viz.: bimetallism and the classification of money under various heads (commodity, token, fiat etc.) Both of course come in for incidental mention; but neither, it seems to me, belongs in a one-quarter course here and now. Bimetallism, as discussed in the books, is today rather a historical relic; the effective issue is not double versus single standard, but gold standard versus “paper standard”. Systematic classification of money has distinct uses; but I’m inclined to trust to the passage on the subject in Robertson and avoid wasting class time on it. The necessary discussion of monetary history affords opportunity to talk a bit about both these matters; but a student who had taken the course with me could probably not explain the relation (e.g.) between over-valued and under-valued metal, or between proper bimetallic and limping standards.

More serious than these omissions of subject-matter is another type of omission: the course as I give it makes little attempt to inoculate the student against cranks by systematic study and dismemberment of the work of theoretical bunglers. It is a rather serious pedagogical defect to include little reading with which the instructor thoroughly disagrees. My excuses firstly that the course must take up more material then can be so treated in a single quarter and secondly that it seems to me the line between correct and incorrect analysis is fairly clearly drawn in the elementary phases of monetary and banking theory here treated. But frankly I’m afraid I must lay more weight on the first excuse than the second.

Albert G. Hart.

 

 

Source: Columbia University Libraries, Manuscript Collections. Albert Gailord Hart Papers: Box 61, Folder “Assignments and Other Memoranda for Reserve in Harper Reading Room/Sec 2 Ec 230 1933 Chicago, Money (Summer Quarter)”.

Image Source:  ditto.

Categories
Chicago Economists Exam Questions

Chicago. Economic Theory Ph.D. Qualifying Exams, 1932-33

In the papers of economist Albert G. Hart at Columbia University there is a folder that contains nearly a complete run of economic theory qualifying exams from the University of Chicago covering the period 1926-1940. I include here the exam from the Spring quarter 1932 and the exam from the Autumn Quarter 1933, though I cannot say whether Hart himself actually took either one of these two theory exams. The previous two postings have field exams (money and banking exam, economic history exam)  that are (i) unique in his papers and (ii) have his handwritten notations, e.g. questions checked and time started and ended for some questions, so we can be very sure those were indeed “his” exams. In several of the theory exams before the Autumn 1933  there are Hart-like checkmarks over the names of economists explicitly mentioned which has led me to conclude that a part of Hart’s personal examination prep was to go over the old theory examinations to identify the economists most likely to make an appearance in his own economic theory exam. The Autumn 1933 exam of this posting has no such checkmarks and would coincide with the quarter he took his money-and-banking exam. In any event today’s postings are still valuable artifacts from the early 1930s Chicago department.

________________________________

ECONOMIC THEORY
Written Examination for the Ph.D.

Spring Quarter, 1932

Time: 3 1/2 hours.

Answer seven questions, of which at least three must be in Part I. C. & A. students may substitute question 6, Part II, for any other question.

Part I

  1. Discuss the relationships between the conclusions and assumptions of the neoclassical school[], the Weber[]-Sombart[] school, and the American institutionalists[].
  2. Trace the development of the demand concept from Adam Smith to the present, touching on the contributions of J.S. Mill[], Cournot[], Fleeming Jenkin[], Walras[], Böhm-Bawerk[], and the statistical economists. [(Schultz)]
  3. A producer of cement has a monopoly of the market in the area adjoining his plant, but is an insignificant factor in the rest of the country, where there are many competing producers. He can sell any desired portion of his output in the competitive market at the price there prevailing. Given the price prevailing in the competitive market, the demand schedule in his own monopolized market, his own average cost schedule, and any additional information which may be necessary for the solution of the problem, find the price he should charge in his own market, and the quantities he should sell in each market, to maximize his net revenue.
  4. Answer (a) or (b), but not both.

(a) The final degree of utility curves of A and B for corn (X) and beef (Y) are as follows, the small letters x and y representing the quantities of X and Y consumed by the person indicated by the subscript.

Commodity

Person

X (corn)

Y (beef)

A

fa(xa) = – (3/2)xa + (19/2)

?a(ya) = -(1/2)ya + 6

B

fb(xb) = -(3/8)xb + 5

?b(yb) = – yb + 7

The total market supply of corn is

x = xa + xb = 14

and the total market supply of beef is

y = ya + yb = 8

Without performing any numerical computations, explain how to deduce the combined demand curves of A and B for corn in terms of beef and for beef in terms of corn.

(b) Is there an equilibrium price and output when a commodity is produced by two competing monopolists? Discuss this problem touching on the solutions of Cournot[], Edgeworth[], Amoroso[], and Wicksell[].

Part II

  1. Describe the history and status of the real cost theory [✓] of value. [Marx]
  2. Point out the resemblances and the differences between the preconceptions, the methods of analysis, and the conclusions, of Adam Smith and the physiocrates [sic], or of the mercantilists and the physiocrates [sic], or of Malthus and Ricardo.
  3. Give some reasonable objectives for a centrally planned economy in a democratic state; state the grounds of your selection of objectives; indicate and discuss possible lines of procedure for realizing them through price control.
  4. Explain and comment on the following in connection with interest theory; [BB; Hayek; Fisher[?]]

(1)  length of the productive period; (2) underestimate of the future; (3) marginal physical productivity of waiting; (4) marginal abstinence; (5) “evening out the income stream.”

5.  Discuss the significant of variability of the proportions of the factors of production and of variability of the supplies of the productive factors for a marginal productivity theory of distribution.

For C. & A. students only

6.   Discuss the feasibility and merits of inflation in the present stage of the depression.

 

Source: Columbia University Libraries, Manuscript Collections. Albert Gailord Hart Collection. Box 60, folder “Exams: Chicago”.

_____________________________________________

ECONOMIC THEORY
Written Examination for the Doctorate

[Part I, Price theory/Microeconomics]
Autumn Quarter, 1933

Time: Three Hours.

Answer all the questions as directed.

1.   (Answer both parts)

A.  Defined or very briefly describe:

(1) Inelastic demand
(2) Elastic demand
(3) Incremental (or marginal) revenue
(4) Perfect competition (in terms of demand elasticity)
(5) Pure profit
(6) Productivity (incremental or marginal of a particular agency or factor)

B.  Is export dumping evidence of domestic monopoly? Explain. Under what conditions does export dumping lead to a lower domestic price in the exporting country?

2.   (Answer either A or B)

A.  State briefly the doctrine of market price and natural price of the early classical economists; contrast this with Marshall’s analysis of long-run and short on price, and give your own view of the correct classification of viewpoints with respect to time.

B. State and critically discuss the classical doctrine of productive and unproductive labor, and in view of the issues raised formulate a correct definition of production in economics.

3.  The theory of marginal utility: its origin, principal forms or interpretations, your own view of its meaning and use in price theory, and the critical appraisal of its validity. Consider especially the relations between the use of the principle as an explanatory concept and as a premise for the discussion of social policy.

4.  (Answer either A or B)

A.  Discuss the effects of establishing by legal action be minimum wage above the wage actually received by, say, one-fourths of the workers actually employed: (a) under conditions of prosperity with approximately full employment; (b) under depression conditions with a large volume of unemployment.

B.  Criticized the view that industry fails to distribute sufficient purchasing power to buy its product, resulting in economic on balance.

5. Show graphically the effect of lowering the tariff on sugar. (Assumed domestic and foreign demand and supply curves given, and neglect any disturbances in the balance of international payments.)

6. Briefly characterize and evaluate comparatively what you considered the significant “approaches” or methodologies in economic science. (The following are to be taken as suggestive catch-words: classical, inductive, institutional, historical, deductive, price theory, sociological, socialistic, control.) We are possible, cite examples of the different tendencies in the history of economic thought from the Greeks to the present.

 

PART II
MONETARY AND CYCLE THEORY

Written Examination for the Ph.D.
Autumn Quarter, 1933

Time: 2 hours

Answer four questions, including the first two.

  1. State the classical doctrine of international gold flows and price levels and discuss some recent criticism of this doctrine.
  2. “The primary cause of business depression is the rigidities of the price structure.”  “Through their alternating contraction and expansion of the circulating medium the banks are responsible for the wide swings in industrial activity.” Discuss these statements.
  3. Discuss the theoretical short-comings involved in a policy on the part of our federal government of progressively bidding up the price of gold in foreign markets.
  4. If business recovery came without the assistance of governmental inflation it would be accompanied by an expansion of the circulating medium as a result of the lending operations of the commercial banks. What significant similarities and differences are there between such expansion and (a) government borrowing from the banks in order to finance public works, (b) outright “greenbackism”?
  5. It has been argued that in as much as the demand for capital goods is a derived demand it follows that any voluntary saving will necessarily result in some degree of unemployment. That is to say, the savings will reduce the demand for consumers’ goods, thus reducing the demand for capital goods, and consequently not all the savings will be borrowed; hence unemployment. But the commercial banks, through their power to create circulating medium, make it possible for entrepreneurs to obtain the funds with which to create capital goods without the reduction in consumer demand which comes with saving. Hence the banks furnish a means of escape from the dilemma. Discuss.

Source: Columbia University Libraries, Manuscript Collections. Albert Gailord Hart Collection. Box 60, folder “Exams: Chicago”.

Image Source:  Social Science Research Building (Lecture Hall 1). University of Chicago Photographic Archive, apf2-07482, Special Collections Research Center, University of Chicago Library.